You are on page 1of 367

BAULKHAM HILLS HIGH SCHOOL

2015

HIGHER SCHOOL CERTIFICATE


TRIAL EXAMINATION

Mathematics Extension 1

General Instructions
Reading time 5 minutes
Working time 120 minutes
Write using black or blue pen
Board-approved calculators may be used
Show all necessary working in
Questions 11-14
Marks may be deducted for careless or
badly arranged work

Total marks 70
Exam consists of 11 pages.
This paper consists of TWO sections.

Section 1 Page 2-4 (10 marks)


Questions 1-10
Attempt Questions 1-10
Allow about 15 minutes for this section.
Section II Pages 5-10 (60 marks)
Attempt questions 11-14
Allow about 1 hour and 45 minutes for
this section.
Table of Standard Integrals is on page 11

Section I - 10 marks
Use the multiple choice answer sheet for question 1-10
1.

If O is the centre of the circle, the value of

in the following diagram is:

50

(A) 25
(B) 40
(C) 50
(D) ) 80
2.

The point P divides the interval AB externally in the ratio 3 : 2. If A(-2,2) and B(8,-3) what is the
coordinate of the point P?
(A) -13
(B) -1
(C) 4
(D) 28

3.

How many distinct arrangements of the letters of the word


if the s are separated.
(A) 720
(B) 1800
(C) 2160
(D) 2520

are possible in a straight line

4.

The polynomial
(A) -28

2. What is the value of

has a factor of

(B) -20
(C) 20
(D) 28
5.

The acute angle between the lines 4

2 and

1 to the nearest degree is :

(A) 12
(B) 13
(C) 40
(D) 41
6.

The equation of an inverse trig function drawn below is :


y

3
2

1
2

1
2

7.

(A)

sin

(B)

sin 2

(C)

3sin

(D)

3sin 2

3
2

A particle moving in simple harmonic motion with displacement


9 16
. What is its amplitude (A) and its period (T) ?
(A) A=3 T=

(B) A=3 T=
(C) A=4 T =

(D) A=4 T =

and velocity , is given by

8.
25
(A)

4
sin

(B) sin
(C) sin
(D) sin

9.

is:

The derivative of tan


(A)

(B)
(C) 4 tan
(D)

1|

10. The solution to |2

(B)

(D)

2| is

(A)

(C)

1
1

1
End of Section 1

Section II Extended Response


All necessary working should be shown in every question.

Question 11

(15 marks) - Start on the appropriate page in your answer booklet

Marks

3
a)

Solve

b)

Find

c)

d)

e)

f)

2
2
1

Evaluate lim

sin
3

using the substitution

1
2

2
Find the constant term in the expansion

sin2

(i)

Show that a root of the continuous function


between 0.4 and 0.5.

(ii)

Hence use one application of Newtons method with an initial estimate


of
0.4 to find a closer approximation for the root to 2 significant figures.

Solve sin2

cos

for 0

lies

End of Question 11

Question 12

(15 marks) - Start on the appropriate page in your answer booklet

sin 2

Marks

a)

Evaluate

b)

(i)

From a group of 6 boys and 6 girls, 8 are chosen at random to form a group.
How many different groups of 8 people can be formed?

(ii)

How many of these groups consist of 4 boys and 4 girls?

(iii)

4 boys and 4 girls are chosen and placed around a circle.


What is the probability that the boys and girls alternate?

c)

The rate of change of the temperature (T) of an object is proportional to the difference
between the temperature of the object and the temperature of the surrounding medium(C),
ie

An object is heated and placed in a room of temperature 20 to cool. After 10 minutes its
temperature is 36 . After 20 minutes the temperature is 30 .

d)

is a solution to the differential equation above.

(i)

Show

(ii)

Find the value of

(iii)

What was the temperature of the object when it was first placed in the room?

and the value of

to 3 decimal places.

1
3
1

Prove
!

1 3 5 (2n+1) =

induction.

End of Question 12

for

0 by mathematical

Question 13

a)

The polynomial

8 has roots ,

Marks

and .

Find :
(i)

(ii)

(iii)

b)

(15 marks) - Start on the appropriate page in your answer booklet

if

has a triple root.

PN is the normal to the parabola


4
at the point P (2 ,
). The normal
intersects the line SN which is parallel to the tangent at P. S is the focus of the parabola.
y

N
2

P(ap,
P
2 ap
, )
S

(i)

Show the equation of the normal PN is

(ii)

Find the equation of the line SN.

(iii)

Show that N has coordinates (

(iv)

Find the equation of the locus of N as P moves on the parabola.

2
1

Question 13 continues on the following page


7

2
2

c)

Sand is falling on the ground forming a conical pile whose semi apex angle is 30.
/ .(

The volume of the pile is increasing at a rate of

30

(i)

Show that the volume of the pile is given by:

(ii)

Find the rate at which the height of the pile is increasing when the height of the

pile is 2 metres.

End of Question 13

Question 14

(15 marks) - Start on the appropriate page in your answer booklet

a)

V
10m

100m

tan

A projectile is fired from the ground with an angle of projection given by


and initial velocity V.
It just clears a wall 10 high 100

b)

away. Let acceleration due to gravity be =10ms .

and

(i)

Show that the equations of motion are

(ii)

Find the initial velocity, V of the projectile.

(iii)

At what speed is the projectile travelling the instant it clears the wall?

Copy or trace the diagram below in your exam booklet.


C

Q
R

x
D

is the midpoint of the arc


diameter

and meets

. The radius

meets

at .

is perpendicular to the

at .

(i)

If

, show

2 .

(ii)

Prove

is a cyclic quadrilateral.

Question 14 continues on the following page

2
2

c)

Below is the graph of

ln
y

e+1
e

(i)

Show that the equation of the inverse function is given by

(ii)

Hence find the area of the shaded region above.

End of Paper.
10

Name: ________________________________

Teacher: _______________________________

Class: _______________________________

FORT STREET HIGH SCHOOL


2015

HIGHER SCHOOL CERTIFICATE COURSE

ASSESSMENT TASK 3: TRIAL HSC


Mathematics Extension 1
Time allowed: 2 hours
(plus 5 minutes reading time)
Syllabus
Assessment Area Description and Marking Guidelines
Outcomes

Chooses and applies appropriate mathematical techniques in
order to solve problems effectively
HE2, HE4 Manipulates algebraic expressions to solve problems from topic
areas such as inverse functions, trigonometry, polynomials and
circle geometry.
HE3, HE5 Uses a variety of methods from calculus to investigate
HE6
mathematical models of real life situations, such as projectiles,
kinematics and growth and decay
HE7
Synthesises mathematical solutions to harder problems and
communicates them in appropriate form

Total Marks 70

Section I
10 marks
Multiple Choice, attempt all questions,
Allow about 15 minutes for this section
Section II 60 Marks
Attempt Questions 1114,
Allow about 1 hour 45 minutes for this section

General Instructions:

Section I

Total 10

Q1Q10

Questions
110
11, 12
13
14
Marks

Section II

Total 60

Marks

Q11

/15

Q12

/15

Q13

/15

/15

Percent

Questions 1114 are to be started in a new booklet.


The marks allocated for each question are indicated. Q14
In Questions 11 14, show relevant mathematical
reasoning and/or calculations.
Marks may be deducted for careless or badly

arranged work.
Board approved calculators may be used.


1 |P a g e

SECTION I (One mark each)


Answer each question by circling the letter for the correct alternative on this sheet.
Allowabout15minutesforthissection.
1

Whichexpressionisacorrectfactorisationof
4
16
(A) (
4
8
16
(B) (
4
(C) (
4
4
16
4
16
(D) (
4

Whichexpressionisequalto

(A)

sin 3

(B)

sin 3

(C)

sin 6

(D)

sin 6

64

Whichinequalityhasthesamesolutionsas
|
2 | |
3 | 5 ?

(A)


6 0

(B)

(C)

| 2

1|

(D)

0
5

AMathematicsdepartmentconsistsof5femaleand5maleteachers.Howmany
committeesof3teacherscanbechosenwhichcontainatleastonefemaleandonemale?
(A)
100
(B)
120
(C)
200
(D)
2500

Considerthefunction

(A)
(B)

(C)

(D)

anditsinversefunction

.Evaluate

3 .

3|P a g e


6 Whichgroupofthreenumberscouldbetherootsofthepolynomialequation


41
42 0?
(A) 2,3,7
(B) 1,6,7
(C) 1,2,21
(D) 1,3,14

7 Afamilyoftenpeopleisseatedrandomlyaroundacirculartable.Whatistheprobability
thatthetwooldestmembersofthefamilysittogether?

(A)

! !

(B)

! !

(C)

! !

(D)

! !
!

8) Let x 1 be a first approximation to the root of the equation cos x log e x .


What is a better approximation to the root using Newtons method?
(A)
1.28
(B)
1.29
(C)
130
(D)
1.31

9 Whatisthevalueof 3
6

sec 2 x
dx ?Usethesubstitution u tan x .
tan x

0.6009

(A)
(B)

0.6913

(C)

log e 3

(D)

log e 3

4|P a g e

10

Let| |

1. Whatisthegeneralsolutionof

(A)

(B)

(C)

(D)

, isaninteger

, isaninteger

, isaninteger

, isaninteger

Question11 (15marks)

a) Evaluate lim

UseaNEWwritingbooklet.

b) Find

c) Find

d) Findtheacuteanglebetweenthelines3

e) Thepoints 2 ,
and 2 ,

(i) Theequationofthechord

is

9.

lieontheparabola

8,and

.(DoNOTprovethis).

Ifthechord passesthrough 0, ,showthat


1.

(ii) Giventhechord passesthrough 0, andthenormalsat and intersectatthe


point whosecoordinatesare

,
2 .

Findtheequationofthelocusof .

5|P a g e


f)

Thesketchshowsthegraphofthecurve
Theareaunderthecurvefor0

(i) Findthe intercept.

(ii) Findthedomainandrangeof

2 cos

where

3isshaded.

2 cos

(iii) Calculatetheareaoftheshadedregion.

Question12 (15marks)

UseaNEWwritingbooklet

a) Let , , betherootsoftheequation
3
6
1 0.

(i) Find2
2
2 .

1
(ii) Find
+

b) Aparticlemovesinastraightlineanditspositioninmetresatanytime secondsisgiven
by
3 cos 2 4 sin 2

(i) Expressthemotionintermsof cos


.

2
(ii) Findtheparticlesgreatestspeed.(Answertothenearestwholenumber).
2

6|P a g e


c)

Acoffeemakerhastheshapeofadoublecone60cmhigh.Theradiiatbothendsare4cm.
/ .
Coffeeisflowingfromthetopconeattherateof5

(i) Showthatradius( inthebottomconeis

(ii) Howfastisthelevelofcoffeeinthebottomconerisingattheinstantwhenthecoffeeinthis
coneis6cmdeep?

3
d) Inthediagram, istangenttoboththecirclesat .
Thepoints and areonthelargercicles,andtheline isatangenttothesmaller
circleat .Theline intersectsthesmallercircleat .

Copyortracethediagramintoyouranswerbooklet.
i) Explainwhy AXD ABD XDB
ii) Explainwhy AXD TAC CAD
iii) Henceshowthat bisects BAC

1
1
2

7|P a g e

Question13 (15marks)

UseaNEWwritingbooklet

a) Inabagthereare6red,4whiteand3blackballs.Threeballsaredrawnsimultaneously.Whatis
theprobabilitythattheseare:

(i) allred.
1

(ii) exactly2whiteballs.
1

b) Inthediagram, , isapointontheunitcircle

1 atanangle fromthe
positive

axis,where

1at .Thepoints 0 ,

.Thelinethrough 0,1 and intersectstheline


and 0 , 1 areonthe

axis.

and

(i) Usingthefactthat

showthat

aresimilar,

sec +tan .

(ii) Showthat

= sec +tan .

(iii) Showthat SNP

(iv) Hence,showthattan

8|P a g e

c) Afreshlycaughtfish,initiallyat18C,isplacedinafreezerthathasaconstant
unknowntemperatureof .Thecoolingrateofthefishisproportionaltothedifference
betweenthetemperatureofthefreezerandthetemperature ,ofthefish.
Itisknownthat satisfiestheequation

where isthenumberofminutesafterthefishisplacedinthefreezer.
(i) Showthat =

satisfiesthisequation.

(ii) Ifthetemperatureofthefishis10 after7 minutes,


Showthatthefishstemperatureafter minutesisgivenby

T x (18 x)e

log
15

10 x t
18 x

(iii) Findthetemperatureofthefishafter15minuteswhentheinitialfreezertemperatureis
5 .Answertothenearestdegree.

d) Usetheprincipleofmathematicalinductiontoshowthat

4
1 7 0forallintegers 2 .

9|P a g e


Question14 (15marks)

UseaNEWwritingbooklet

a) Fromapoint isduesouthofatower,theangleofelevationofthetopofthetower , is23.


Fromanotherpoint ,onabearingof120,fromthetower,theangleofelevationof is32.
Thedistance is200metres.

i)

ii)

Copyortracethediagramintoyourwritingbooklet,addingthegiveninformationtoyour
diagram.

Hencefindtheheightofthetowertothenearestmetre.

b) AparticleisprojectedhorizontallyfromapointP, metresabove ,withavelocityof metres


persecond.Theequationofthemotionoftheparticleare
=0and = .

(i)

Ifthehorizontalpositionoftheparticleattime is
isgivenby

,showthattheverticalposition

10|P a g e

Acanistercontainingaliferaftisdroppedfromahelicoptertoastrandedsailor.The
helicopteristravellingataconstantvelocityof216km/h,ataheightof120metresabove
sealevel,alongapaththatpassesabovethesailor.

Howlongwillthecanistertaketohitthewater?(Answertoonedecimalplace).

2
(Takeg 10 / ).

Acurrentiscausingthesailortodriftataspeedof3.6km/hinthesamedirectionasthe
planeistravelling.Thecanisterisdroppedfromtheplanewhenthehorizontaldistance
fromtheplanetothesailoris metres.Whatvaluescan takeifthecanisterlandsat
most50metresfromthestrandedsailor?

(ii)

(iii)

c) Thedepthofwater metresonatidalcreekisgiven
thetimebeingmeasuredinhours.

(i)
Drawaneatsketchof

4 cos ,for0

4 .

4 cos ,showingallimportantfeatures.

Ifthelowtideonedayisat1.00p.m.,whenistheearliesttimethatashiprequiring3mof
watercanenterthecreek?Giveyouranswerinhoursandminutes.

(ii)

END

11|P a g e

Girraween High School


2015 Year 12 Trial Higher School Certificate

Mathematics Extension 1
General Instructions
Reading tjmc - 5 mjnutcs

Working time- 2 hours

Total marks - 70

a11

( Section I

l
1,

Write using black or blue pen


Black pen is preferred

i:

10 marks

!J

Boa.rd-approved calculators may


be used

Attempt Questions 1-10


Allow about 15 minutes for this section

A table of standard integrals is


provided at the back of this paper
In Questions 11-14, show
relevant mathematical reasoning
and/or calculations

>I

IIq
11

( Section II )
60 marks
Attempt Questions 11-14
Allow about 1 hour and 45 minutes for this section

For Section II: Questions 11- 14 MUST be returned in clearly marked separate sections.
On each page of your answers, clearly write:
~

the QUESTION being answered

YOURNAME

your Mathematics TEACHER'S NAME.

Start each new question on a NEW PAGE.


You may ask for extra pieces of paper if you need them.

For questions 1-10, fill in the response oval corresponding to the correct answer
on your Multiple choice answer sheet.
1. What is the acute angle between the lines y

= 2x -

3 and 3x + 5y -1 = 0 , to the

nearest degree?
A)32

B) 50

D) 86

C)82

2. The number of different arrangements of the letters of the word REGISTER which
begin and end with letter R is:

A)

6!
(2!)'

B) 8!
2!

D) 8!
2!2!

C) 6!
2!

3. The middle tenn in the expansion (2x - 4)4 is


A) 81

B) 216x 2

C) 384x

D) -96x 3

4.

,\' l'{XJ

Which of the following could be the polynomial y = P(x)?


A) P(x)=x 3 (2-x)

B) P(x)

= x' (2-x) 2

C)P(x) =x 3 (x-2)

D) P(x)

= -x 3 (x + 2)

5. The coordinates of the points that divides the interval joining


(-7,5) and (-1,-7) externally in the ratio 1:3 are
A) (-10,8)

B)(-10,11)

C) (2,8)

D)(2,11)

!'..
8

6. Which of the following represents the exact value off cos' xdx?
0

A) re-2.J2

B) re-2.fi.

16

C) re+ 2.J2

16

7. Which of the following represents the derivative of y = cos-' (

I
A)----

x~x'-I

B)

-1
~x 2 -1

D) re+ 2.J2

C)

1
~x 2 -1

!)

D)

x~x 2 -I

. he value o fI- + I + -I ?
8. Let a,,B,ybe the roots of 2xi+,
x -4x+ 9 = 0 .What 1st

a,B

D)

,By

ay

__!__

9. If cose = _I and O< e <re, then tan~ is equal to:


5
2
-1
A) -or 3
3

1
B) -or 3
3

C) -2

D)2

I 0. A particle is moving in Simple Harmonic Motion and its displacement, xunits,


at time t seconds is given by the equation x = A cos(nt) + 2 .The period of
the motion is 4re seconds and the particle is initially at rest,12 units to the right
of the origin. Find the values of A and n .
I
A)A=IOn=,
2

B)A=I0,n=2

I
C)A=l2,n=2

D) A=12,n=2

Marks

Questionll.(15 marks)-show all necessary working)


5
x-1

(a)Solvefor x: - > 2

(b) Find the value of e, such that

-/3 cos e - sine = 1, where

f sin2x

O:;:; e:;:; 2:r. 3

'3

. . u = sm
. ,- x to evaluate
(c) U se t h e sub st1tut10n

l+sm 2 x

dx

'4
3

Give your answer in simplest form.

(d) Use the mathematical induction to show that for all positive
integers n 2:: 2,
2x1+3x2+4x3+ ............... +n(n-1)=

(e) The coefficients of x' and

x-'

n(n 2 -1)
3

in the expansion of

(ax-:,)'

are the same, where a and b are non-zero. Show that a+ 2b = O.

Question 12.(15 marks)

a) i) Find !!_cos-'
dx

(x -IOI 0 J.

10

ii) Hence, evaluate

f-J
5

1
20x-x 2

b) Two points P(2ap,ap 2 ) and Q(2aq,aq 2 ) lie on the parabola x 2

= 4ay.

The general tangent at any point on the parabola with parameter t is given by

= tx- at 2

i)

(DO NOT prove this).

Find the coordinates of the point of intersection T of the tangents to the


parabola at P and Q.

You are given that the tangents at P and Q intersect at an angle of 45.

ii)

Show that

p - q = 1 + pq

By evaluating the expression x' - 4ay, or otherwise, find the locus of the

iii)

point T when the tangents at P and Q meet as described in part (ii) above.

c) The velocity v

111

Is of a particle moving in simple hannonic motion along

the x-axis is given by v2

i)

ii)

= 8 + 2x- x'.

Between what two points is the particle oscillating?

. What is the amplitude of the motion?

iii)

Find the acceleration of the particle in tenns ofx.

iv)

Find the period of oscillation.

Question 13.(15 marks)

a) Let ABPQC be a circle such that AB= AC, AP meets BC at X and AQ


meets BC at Y as shown below. Let LEAP= a and LABC

= f3.

A~

\_JP
I

i)

Copy the diagram in your writing booklet, marking the information given
above.

a + fJ .

ii)

State why LAXC

iii)

Prove that L.BQP = a .

iv)

Prove that L.BQA

= f3 .

v)

Prove that the quadrilateral PQYX is cyclic.

b) When the polynomial P(x) is divided by x 2 -1, the remainder is 3x + 1. What


is the remainder when P(x) is divided by x + 1?

c)

A
205 m

NOTTO SCALE
A is 205 metres above the horizontal plane BPQ . AB is vertical. The angle

of elevation of A from P is 37 and the angle of elevation of A from Q


is 22

.P

is due East of B and Q is south 47 east from B . Calculate the

distance from P to Q, to the nearest metre.

d) Four people visit a town with four restaurants A, B, C and D.


Each person chooses a restaurant at random.

i)

Find the probability that they all choose different restaurants.

ii)

Find the probability that exactly two of them choose restaurant A.

Question 14.(15 marks).


a) The graph of y =I+ 2 sin -i (2x -1) is shown in the diagram.

y
,":\
c

I___..........

,,
\/
Determine the values of a, b and c.

Question 14 continues on the next page

b) 1.) Stale th e range o f y = tan -1

ii) Find dy for the function

dx

.Jx2 - 4 .

.Jx2

-4
y=tan-1 - - 2

c) Find the volume of the solid when the region enclosed entirely by the curves
y = sin x and y = sin 2x over the domain Os x

s re
2

is rotated about the

x axis.

d) A projectile is fired from the origin towards the wall of a fort with initial velocity

Vms- 1 at an angle a to the horizontal.

On its ascent, the projectile just clears one edge of the wall and on its decent
it clears the other edge of the wall, as shown in the diagram.
The equations of motion of the projectile are

x = Vt cos a and y = Vt sin a - g t 2 (Do not prove this.)


2

i)

V 2 sin2a
Show that the horizontal range R of the projectile is - - - g

Question 14 continues on the next page

'

ii)

Hence, show that the equation of the path of the projectile is

y=x(1-;}ana.
iii)

The projectile is fired at 45 and the wall of the fort is !Ometres high. Show that
the x coordinates of the edges of the wall are the roots of the equation
x2

iv)

Rx+ I OR= 0.

If the wall of the fort is 4.5 metres thick, find the value of R.

End of examination!!!

STUDENT NUMBER

GOSFORD HIGH SCHOOL


2015
TRIAL HSC EXAMINATION

EXTENSION 1 MATHEMATICS
General Instructions:

Total marks: - 70

Reading time: 5minutes


Working time: 2 hours
Write using black or blue pen
Board-approved calculators may
be used
A table of standard integrals is
provided
In Questions 11-14, show relevant
mathematical reasoning and/or
calculations

Section I (10 marks)


Attempt Questions 1- 10.
Answer on the Multiple Choice answer
sheet provided
Allow about 15 minutes for this section
Section II (60 marks)
Attempt Questions 11-14
Start each question in a separate answer
booklet
Allow about 1 hour and 45 minutes for
this section

MULTIPLE CHOICE

/10

QUESTION 11

/15

QUESTION 12

/15

QUESTION 13

/15

QUESTION14

/15

TOTAL

/70

Section I.
Total marks (10).
Attempt Questions 1-10.
Allow about 15 minutes for this section.
Answer on the multiple choice answer sheet provided. Select the altemative A, B, C, D that
best answers the question. Fill in the response oval completely.

1. The point A has coordinates ( -1,4) and the point B has coordinates (5, -2). Find the
coordinates of the point which divides AB externally in the ratio 1:3.

A. (-4, 7)

C. (7,-4)

B. (4, -7)

D. (-7,4)

x 2 -2

2. The solution to the inequation - - :5 1 is


x
A.

x :5 -1, x ;;;: 2

c.

x :5 -1, 0

< x :5 2

B.

-1:5x<O, x;?:2

D.

x :5 -1, 0 :5 x :5 2

3. A committee of three is to be chosen from a group of five men and seven women.
How many different committees can be fo11ned if the committee is to contain at least
one man and at least one woman?

A.

220

B.

c.

175

70

105

D.

4. If the acute angle between the lines 2x - y == 2 and kx - y == 5 is 45, then the
value of k is
-1

-1

B. -3 or -

A. 3 or -

C. 3 or -

D. -3 or -

5. The acceleration of a paiiicle moving along a straight line is given by a == -ze-x


where x metres is the displacement from the origin. If the velocity of the paiiicle is
given by v, then

A. v 2 == ze-x

+c

B. v 2 == 2ex

+c

C. v 2 == 4e-x

+c

D.

v 2 == 4ex + c

1
f 1+9x

A.

27
1

c.

dx=
1

B. - tan- 1 3x

tan- 1 3x + c

D. tan- 1 3x + c

- tan- 1 3x + c
3

dN
dt

= -0.4(N -

7. If -

100) and N

+c

= 0 when t = 0, the value of N correct to 2

decimal places when t = 20 is


B. 99.97

A. 7.69

C. 100.03

D. 192.31

8. Victoria made an error proving that 32" -1 is divisible by S (where n is an integer


greater than 0) using mathematical induction. Part of the proof is shown below.

Step 2: Assume the result true for n = k


32 k -1 = SP where P is an integer.
2

Hence 3 k

Line 1

= SP+ 1

To prove the result is true/or n =k+ 1


32 <k+1l -1 = SQ where Q is an integer.

Line 2

LHS = 32 <k+iJ -1

= 32k x3 2 -1
= (SP+l)x3 2 -1

Line 3

=72P+l-1

Line4

=72P

= S(9P)
=SQ

= RHS
Which line did Victoria make an enor?

(A) Line 1

(B) Line 2

(C) Line 3

(D) Line 4

9.

Which of the following is an expression for f 2cos 2 x dx.


A.

1
. 2
x--sm
x+c
2

C.

x - sinZx + c

D.

x + sinZx + c

10. One approximation to the solution of the equation

"+ tan' x-x 2 =O is x =1. What

4
is another approximation to this solution using one application ofNewton's method?

A. x == 1.3805

B. x == 1.3914

c.

x == 1.4125

D. x == 1.4156

Section II.
Total marks (60).
Attempt Questions 11-14.
Allow about 1 hour and 45 minutes for this section.
Answer all questions, starting each question in a separate writing booklet.

Question 11 (15 marks) Use a SEPARATE writing booklet.


d
Find- (x sin 2x).
dx

(a) (i)

(ii)

(1)

Hence or otherwise find

f x cos 2x dx .

(b) Consider the function f(x) = 2 cos- 1

(3)

-.

(i)

Evaluate f(O).

(ii)

Draw the graph of y =

(iii)

State the domain and range of y = f(x).

(1)

f (x).

(c) If ex,~ and y are the roots of 2x 3

6x 2

(1)

+x +2 =

(2)

0, find the value of

(i)

a+ p + y.

(1)

(ii)

(a - 1)(/3 - 1)(y - 1).

(2)

(d) Evaluate

f16 x-Jx + 3 dx

by means of the substitution u 2 = x

+ 3.

(4)

Question 12 (15 marks) Use a SEPARATE writing booklet.


(a) Two points P(2ap, ap 2 ) and Q(2aq, aq 2 ) lie on the parabola x 2 = 4ay.
(i)

Show that the equation of the tangent to the parabola at P is


y

= px-ap

(2)

(ii)

The tangent at P and the line through Q parallel to the y axis intersect
(2)
at T. Find the coordinates ofT.

(iii)

Write down the coordinates of M, the midpoint of PT.

(1)

(iv)

Determine the locus of M when pq = -1.

(1)

(b) The diagram below shows a cyclic quadrilateral MNKL with MN II LK.

p
NOTTO SCALE

PN is a tangent to the circle and LMNK = 2LKNP.

Copy the diagram into your writing booklet and prove that t,.LMK is isosceles.
Hence, show that MK bisects LLMN.

(4)

(c) The point P(Z;-1) divides the interval joining A(-2,3) and 8(8, -7) internally
the ratio m: n. Find the values of m and n.
(3)

(d) Differentiate

(2)

Question 13 (15 marks) Use a SEPARATE writing booklet.


dv
Showthat-

d
1
=
(
v
dt
dx 2

(a) (i)

2 ).

(2)

(ii)

A particle is moving along a straight line. At time, t seconds, its displacement,


x metres, from a fixed point O on the line is such that t = x 2 - 3x + 2. Find
an expression for its velocity v in terms of x.
(I)

(iii)

Hence, find an expression for the particle's acceleration a in terms of x.

Express ...f3cosx - sinx in the form Rcos(x + a) where O :5 a :5

(b) (i)

Tr

2.

Hence, or otherwise, solve ...f3cosx - sinx = 1.

(ii)

(c) How many 4-letter "words" consisting of at least one vowel and at least one
consonant can be made from the letters of the word EQUATION?

(2)

(2)
(2)

(2)

(d) The region bounded by the curve y = cos 2x and the x-axis between x = 0 and
Tr

x = - is rotated about the x-axis. Find the exact value of the volume of the solid of
4

revolution generated.

(4)

Question 14 (15 marks) Use a SEPARATE writing booklet.

(a) Use mathematical induction to prove that for all positive integers n:
n

L r(r!)

= (n + 1)! - 1.

r=l

(4)

(b) A particle moves in a straight line so that its displacement, x metres, at time t
seconds, is given by x = 4 - 2sin 2 t.

(c)

(i)

Show that the motion is simple harmonic.

(2)

(ii)

Find the period and the centre of the motion.

(2)

(iii)

Show that the velocity v of the particle in te1ms of its displacement can be
expressed as v 2 = 4(-8 + 6x -x 2 ).
(2)

(i) Show that the range offligbt of a projectile fired at an angle of a to the
horizontal with velocity v is

v 2 sin2a

where g is the acceleration due to

gravity.

(2)

The equations describing the trajectory of the projectile are:

= vt cos a, y = vt sm. a -

-12 gt 2 .

(You are NOT required to prove these equations)

(ii) A cannon fires a shell at an angle of 45 to the horizontal and strikes a point
SOm beyond its target. When fired with the same velocity at an angle of 30 it
strikes a point 20m in front of the target. Calculate the horizontal distance between
the cannon and the target co11ect to 2 decimal places.
(3)

END OF PAPER

2015
Assessment Task 4
Trial HSC Examination

Mathematics Extension 1
Examiners - Mrs D. Crancher, Mrs S. Gutesa, Mr S. Faulds, Ms P. Biczo

General Instructions

Total marks (70)

II

!section

o Reading Time - 5 minutes


o

Total marks (10)

Working Time - 2 hours

o Write using a blue or black pen.

Attempt Questions 1 - IO

Board approved calculators and


mathematical templates and instruments
maybe used.

o Answer on the Multiple Choice


answer sheet provided on the
last page of this question
booklet.

Show all necessary working in


Questions 11,12,13 and 14

o Allow about 15 minutes for this


section

o This examination booklet consists of 13


pages including a standard integral page
and a multiple choice answer sheet.

lsection

nl

Total marks (60)


o

Attempt questions 11 to 14

Answer each question in the


writing booklets provided.

Start a new booklet for each


question with your student
name and question number at
the top of the page.

All necessary working should


be shown for every question

o Allow about I hour 45 minutes


for this section

Student N a m e : - - - - - - - - - - - - - - - - - - Teacher: - - - - - - - - - - - - - - - - - - -

Section I
10 marks
Attempt Questions 1 - 10
Allow about 15 minutes for this section
Use the multiple choice answer sheet for Questions 1 - 10.

The solution to the inequality x( 2- x )( x + 1) ::0: 0 is


(A)

x S: -2 or OS: x S: 1

(C)

x:S:-1 or 0:S:x:S:2

(B)
(D)

-2 S: x S: 0 or x ::0: I
-1 S: x S: 0 or x ;:,: 2

A committee of3 men and 3 women is to be fonned from a group of8 men and 6 women.
How many ways can this be done?
(A)

48

(B)

1120

(C)

40320

(D)

3003

c
NOTTO SCALE

In the diagram, AB is a tangent to the circle, BC= 6cm and CD


What is the length of AB?

(A)

6Ji cm

(B)

6./3 cm

(C)

72cm

(D)

108cm

= 12cm.

What is the equation of the tangent at the point ( 4 p, 2 p 2 ) on the parabola x


(A)

y=px-p'

(B)

x+py=2p+p 3

(C)

x+ PY =4p+ p'

(D)

= 8 y?

= px-2p 2

Hur/stone Agricultural High Schoo/ 2015 Trial HSC Mathe111atics Extension 1 Exa11li11atio11

Page 3

What is the acute angle to the nearest degree that the line 2x-3 y + 5 = 0 makes with the y-axis?

(A)

27

(B)

34

(C)

56

(D)

63

Which of the following statements is FALSE.

(A)

cos- 1(-8) = -cos- 1

(C)

tan- 1 (-8) =

(C)

tan-I fJ

(B)

sin-I (-8) = -sin-I fJ

(D)

cos- 1 (-8) = n - cos- 1 8

(B)

-3 ( 3x-' -I )' +c

(D)

-2x ( 3x 2 -I )5 +c

The ptimitive of 2x( 3x 2 - I)' ts:

(A)

1 ( 3x' -1 )' +c

15

2x

(3x -l) +c

15

The equation(s) of the horizontal asymptote(s) to the curve y =


(A)

y=O

(B)

x=I

(C)

y=l

(D)

x=l only

x:x- +-11 are

What are the coordinates of the point that divides the interval joining the points A(2, 2) and
B(4,5) externally in the ratio 2:3?
(A)

(-2,-4)

(B)

(-2,11)

(C)

(8,-4)

(D)

(8,11)

Hurlstoue Agricultural High School 2015 Trial HSC Mathe111atics Extension I Exa111inatio11

Page 4

10

Which of the following equations is shown in the sketch below

2TI
3

-,
3n
--

-n:

3n
2

IT

--2

l{

--

'

.}

2TI
.-

-TI

(A)

.
)
= cos -I ( smx

(B)

= sin- 1 (cosx)

(C)

= sin -i (x) + sin(x)

(D)

= cos 1 (x) + cos(x)

- End of Section I -

Hur/stone Agricultural High School 2015 Trial HSC Mathentlllics Extension I Exa111inatio11

Page 5

Section II
60 marks
Attempt Questions 11 to 14
Allow about 1 hour 45 minutes for this section
Answer each question in the appropriate writing booklet.
All necessary working should be shown in every question.

Marks

Question 11 (15 marks)


>l?

(a)

Solve the inequality

(b)

In what ratio does the point (14,18) divide the interval joining

x(2x-1)

X (-1,3)to Y (4,8)?

(c)

(i)

Show that the curves y


at the point (-2,

(ii)

(d)

(i)

(ii)

(e)

=x 3 -

x and y

=x -

x' intersect

-6)

Dete1111ine the acute angle between the curves y

=x3 -

and y = x - x' at the point of intersection, to the nearest minute.

A class of25 students is to be divided into four groups consisting


of 3, 4, 5 and 6 students. How many ways can this are done?
Leave your answer in unsimplified fonn.

Assume that the four groups have been chosen.


How many ways can the 25 students be arranged around a circular
table if the students in each group are to be seated together?
Leave your answer in unsimplified fonn.

Five different fair dice are thrown together.


What is the probability the five scores are all different?

Hur/stone Agricultural High Schoo/ 2015 Trial HSC Mathe1natics Extension 1 Exa111i11utio11

Page 6

Marks

Question 12 ( 15 marks)

(a)

Consider the function

l(x) = (x - I)'

= l(x).

(i)

Sketch y

(ii)

Explain why

l(x)

does not have an inverse function for

all x in its domain.


(iii)

State a domain and range for which


function

l(x)

has an inverse

1-' (x).

x;:,: I

1-' (x).

(iv)

For

(v)

Hence , on a new set of axes, sketch the graph of y

find the equation of the function

(b)

. d
F111

(c)

Find the exact value of tan( 2 tan_,

(d)

Find the general solution to 2 cos x

dx

= 1-' (x).

.J9-4x 2

ij

-Ji.

Leave your answer in terms of re .

(e)

Differentiate (with respect to x)

and hence find the exact value of


JJ tan-'.:'.'.

3 Ix
x +9
2

Hur/stone Agricultural High School 2015 Trial HSC Mathe111atics Extension I Exa111inatio11

Page 7

Marks

Question 13 (15 marks)

(a)

The points P ( 2ap, ap') and Q (2aq, aq 1 ) lie on the parabola x 2

= 4ay

such that OP is perpendicular to OQ.


y

P(2ap, a/)

(i)

Prove that pq = -4.

(ii)

R is the point such that OPRQ is a rectangle.

Explain why the co-ordinates of R are ( 2a (p + q), a ( p' + q


(iii)

(b)

(c)

)).

Show that the locus of Risa parabola.

Find by division of polynomials, the remainder when x' + 4 is


divided by x-3.

a, f3 and y aretherootsoftheequation x3 -3x'-6x-1=0.


' p-+r.
a'
'
. d a-+
F111

Question 13 continued next page ....

Hurlsto11e Agricultural High School 2015 Trial HSC Mathe111atics Extension I Exa11d11atio11

Page 8

Question 13 continued ....

(d)

(i)

(ii)

Consider the curve .f(x)=sin 2 x-x+l for 0:S:x:S:ir.


Show that it has one stationary point and detennine its nature.

.f(x)=sin 2 x-x+l has a zero near x, =

ir.

Use one application of Newton's method to obtain another


approximation x 2 , to this zero.

(iii)

rr/2

The graph off ( x) = sin 2 x-x + 1 is shown in the vicinity of x =

ir.

By using this diagram, detennine if x 2 is a better approximation


than x1 to the real root of the equation. You must justify your
answer.

Hur/stone Agricultural High School 2015 Trial HSC Mathe111atics Extension I Exa111i11ation

Page 9

Marks

Question 14 (15 marks)


(a)

In the diagram above, FG is a common tangent and FBIIGD.

(b)

(i)

Prove that FAIIGC.

(ii)

Prove that BCGF is a cyclic quadrilateral.

(i)

Find:

_:'.._ ( x sin 3x)


dx

,T

f
6

(ii)

Hence, evaluate:

xcos3xdx

(c)

Use the substitution y =

f
(d)

Fx to find

dx
~x(l-x)

Use mathematical induction to prove the inequality:

n ! > 2", for all positive integral values of n;:: 4

- End of Section II -

Hur/stone Agricultural High School 2015 Trial HSC Mathe111atics Extension I Exa111i11atio11

Page 10

Year 12 Mathematics Extension I T1ial 2015


Question No. 11

Solutions and Marking Guidelines

Outcon1es Addressed in this Question


PE3 Solves problen1s involving pem1utations and co1nbinations, inequalities and polynon1ials.
HS Ann lies annropriate techniaues from the studv of geo1netry.

Outcome

Markin<' Guidelines

Solutions
(a)

3 marks
Co1Tect solution
2 marks

PE3

--->!
x(2x I)

Substantial progress to\vards

Multiply by the square of the denominator

con-ect solution
1 n1a1k

3x(2x - 1) > x 2 (2x -1 )2

Saine progress to,vards

coJTect solution

3x(2x-l)-x (2x-1) > 0


x(2x-1)(3-x(2x-l)) > 0
x(2x-1)(-2x 2 +x+3) > 0
-x(2x-1)(2x-3)(x+l) > 0

I
3
:.-l<x<O, -<x<-

!I

.,

;\ I\

25

(b)
A(-1,3) B(4,8) P(l4,18)

H5

Ratio AB: BP, m: 11 assume 1: k


.Xo

n1x,

+ nx1

111 +n

](4)+/c(-1)
l+k
14+ 14/c = 4-k

2 111arks
Conect solution.
1 mark
Substantial progress to,vnrds
correct solution.

14

-2

k=3
-2

l:k=l:3

:. Ratio is -3:2
(c)(i)
Substitute x = -2 into y = x 3 - x

HS

RHS = (-2) 3 -(-2)


=-6

: . Satisfies the curve.


Substitutex= -2 into y = x-x'
RHS = -2-(-2) 2
=-6

:. Satisfies the curve.


:. (-2, -6) is the point of intersection.

1 mark

CotTect solution

(c)(ii)

For y

= x 3 -x, y'=3x2 -1

when x = -2, 111, = 3(-2)" -1 = 11


Fory=x-x

y'=l-2x

whenx=-2,1112 =1-2(-2)=5
tane

1111 -1112

3 marks
C01Tcct solution \Vith co1Tect
rounding

. 2 marks
Substantial progress to,vards
co1Tect solution

1 n1ark
Some progress to,vards
correct solution

l+m1m2

11-5 ,
tan e = ,____
I+ (11)(5)
6
)
56
:. e = 67' (to the nearest minute)

:.B=tan- 1 (

PE3

25!
or----3!4!5!6!7!

Substantial progress towards


correct solution

(d) (ii)

2 n1arks

(11-1) ! x 3! x 4 ! x 5 ! x 6 !

Co1Tect solution.

PE3

(e)

HS

2 marks
CotTect solution
l mark

P(E)

6x5x4x3x2
6'
5
54

1 n1ark
Substantial progress towards
cmTcct solution.

2 n1arks
Correct solution.
1 n1ark
Substantial progress to\l1anls
correct solution.

Task 4 Trial HSC


Mathematics Extension I
Solutions and Marking Guidelines
Outcomes Addressed in this Ouestion
uses the relationship between functions, inverse functions and their derivatives

Year 12 2015
Question No. 12
HE4

Outcome

HE4

Markin!! Guidelines

Solutions
Question 12
a)
(i)

41
I Mark for correct sketch

HE4

'

(ii)
It does not have an inverse because for every y value there is
more than one x value.
Or
Does not pass the horizontal line test.
Or
Anything that is equivalent.

HE4

(iii)
Domain: x;:: I
Range: y ;:: 0

HE4

(iv)
x=(y-1)

I Mark for correct explanation

l Mark for c01Tect answer

2 Marks for complete correct


solution

.J; = v-1

l Mark for paitial correct


solution

y=l+.J;
:.r'(x)=I+.J;

HE4

(v)
I Mark for c01Tect sketch

5
4
3
2
1
0
-4 .3 -2 1
-1

I' ,

-2
I

HE4

(b)

dx

2 Marks for complete correct


solution

../9-4x

-)~
+;"'[f}c

I Mark for paitial correct


solution

or

I sm
.

-(2x)
+C
3

HE4

(c)

3
:. tanB=4

2 Marks for complete correct


solution

now,

tan( 2 tan_,

i)

= tan(2B)

1 Mark for partial correct


solution

2tanB
l-tan 2 B

2(i)
1-(i )'

=--'--'--

24

=-

HE4

(d)

2 Marks for complete correct


solution

-./3
-./3
cosx=2cosx =

1 Mark for pmiial correct


solution

:.x=2111rcos-I --./3
2

Jr

= 2111r-, wheren1sanymteger.

HE4

(e)
d

-1

'
x -

-:(tan -)
d.,

-l

3 Marks for complete correct


solution

I
2(1an -) ----:,

3 [ I+~
9

3 ,)
-( _, x)(
3 9+xtan-' 5J

2 Mark for substantial working


that could lead to a correct
solution with only one error.

=2 tan -

=6 9+x2

l Mark for correctly

no,v,

differentiating !!:_(tan-'

Jj[ tan_,xJ l fJj (tan_,xJ


--?
dt=6 ------}- d,
9+.x
f 9+x6
0

=i [(tanI [( tan

;r]~

_,Jj)'
( _,o)'
)
3
3

=i ((i)'-oJ
216

-tan

d,

!:)3

Year 12 Trial Higher School Certificate


Extension 1 Mathematics
Solutions and Marking Guidelines
Question No. 13

Examination 2015

Outcomes Addressed in this Question


PE3 solves problems involving polynomials and parametric representations
PES determines derivatives which require the application of more than one rule of
differentiation
H6 uses the derivative to determine the features of the graph of a function
HE7 evaluates mathematical solutions to problems and communicates them in an appropriate
form.
Markin!! Guidelines
Outcome
Solutions
PE3

(a) (i) OP

= -1.

2 marks : correct
solution
I mark : significant
progress towards COITect
solution

ap + aq, ap' ; aq' )

2 marks : correct
solution
1 mark : significant
progress towards co1Tect
solution

OQ, :. mOP xm OQ

_l_
2

ap
aq'
:.-x-=-l
2ap 2aq
:. p xi=-1

2 2
:.pq=-4.

(ii) Midpoint PQ =
PE3

As the diagonals bisect one another in a rectangle, OR


will also have the same midpoint as PQ.

If 0(0, 0), R has midpoint ( ap +aq, ap' ;aq'), then


2

Ris (2a(p+q), a(p +q


PE3
(iii)

At

)).

R, 1::::;P + q)
2

y=a(p +q

2 marks : c01Tect

~;]

[3]

From [2], x 2 =4a'(p+q)'


:. x

solution
I mark : significant
progress towards c01Tect
solution

=4a 2 (p 2 +q' +2pq)

Substituting [1] and [3],

x =4a'(; + 2x-4),

:. x 2 = 4a (y-8a), which is a concave up parabola with

ve1iex ( 0, 8a).
(b)

PE3

x+3
x-3)x' + Ox+ 4 x

-3x

Remainder is 13.

3x+4 3x-9
13
(c) From x

2 marks : c01Tect
solution
I mark : significant
progress towards c01Tect
solution

3x - 6x- l = 0,
-b
c
a+/J+r=-=3, afJ+/Jr+aA.=-=-6.
a
a
-

(a+ /3 + r ) = a + /3 + r' + 2 ( afJ +/Jr+ ar)

2 marks : c01Tect
solution
I mark : significant
progress towards correct
solution

:. a'+ /3 2 + r' =(a+ /J + r)' -2(a/J + /Jr+ar)


2

:. a'+ /3 2 + r' =(3) -2(-6)


= 21.
H6, PE5

(d) (i) f(x) = sin 2 x-x+ I


f'(x) = 2sinxcosx-l
:. f'(x)

3 marks : coITect
solution
2 marks : substantial
progress towards COITect
solution
I mark: significant
progress
towards correct solution

= sin2x-l

f'(x) = 0 for stationary points.


Solving sin 2x- l = 0,
sin2x = I
For O:,; x:,; n-, 0 s 2x s 2n-.

.
2
7r
o vmg, x = - ,
SI

:.

Testing x =

f' (x) =sin 2x-l,

for

7r,

f'(x)

7r

7r

J5-2

J5-2

"4

"6

one stationary pomt, at x = -

-2

J5-2 is negative, there is a horizontal point of

As

. fl .
111 ex10n at x

7r
= -.

2 marks : c01Tect
solution
l mark : significant
progress towards c01Tect
solution

. ' n- n- I
n- =sm----+ =2-(11.. ) f (n-)

PE5, HE7

f'(; )=

sinn--1 =-1.
7r

Newton's method: x,
-

7r

=-

:. x, =

2-- __
2

2
2

-1

(iii)

I mark : c01Tect
explanation

HE7
I
I
I

I
n/2

real root
~x,

\~

x, is where the tangent at " meets the x axis.


2
This is closer to the real root, : . a better
approximation.

Year 12 Mathematics Extension 1 Trial Examination 2015


Question No. 14
Solutions and Marking Guidelines
Outcomes Addressed in this Ouestion
PE3
solves problems involving circle geometry
HE2 uses inductive reasoning in the construction of proofs
HE4 uses the relationship between functions, inverse functions and their derivatives
HE6 determines integrals by reduction to a standard form throu!!h a !!iven substitution
Outcome
Solutions
Markin!! Guidelines
PE3
(a)(i) Let LDGH= a
2 marks
..
LBFG = a
(corresponding angles, FBIIGD) Correct solution \Vith full reasoning.
1 mark
No,v, LGCD = a
(angle bet,veen a chord and
Substantial progress to\vards a correct
tangent is equal to the angle in
the alternate segment)

solution.

Similarly,
LFAB = a
Since LGCD = LFAB = a,
FAIIGC (corresponding angles are equal)

(ii) Since LGCD


PE3
also,

(shown above)
(angles on a straight line
(sho,vn above)

LGCB = 180- a
LBFG = a

L.GCB + L.BFG = 180 - a+ a


= 180

2 marks
Correct solution ,vith full reasoning.
1 mark
Substantial progress to,vards a correct
solution.

:. BCGF is a cyelie quadrilateral (opposite angles supplementary)

HE4

(b) (i)

!!... xsin 3x = x.3cos 3x+ sin 3x.l


dx

= 3xcos3x+sin3x

2 marks
Correct application of product ntlc to find
correct answer.
1 mark
Demonstrates kno,vlcdge of product rulc in
making substantial progress to a full
solution.

(ii)

HE4

If ~xsin3x = 3xcos3x+ sin3x

3 marks
Correct solution.
2 marks
Correctly finds the required primitive
function.
t n1ark
Substantial progress to\vards finding the
required pri1nitive function.

dx

then 3xcos3x=~xsin3x-sin3x

dx
I d
.
I .
xcos.x=--xs1n
3
3x--s1n 3x
3dx
3
Integrating both sides,
-n6

-rr6

x cos 3:rdx = .!.


3

-6n

!.!...x sin 3xdx- .!.


dx
3

sin 3xdx

= .!.[xsin 3x+ .!.cos3x]~


3
3
0
=.!.[(!!_sin!!.-+ .!.cos!!.-)-( OsinO + .!.cosO )]
3 6
2 3
2
3

=1[(~-o )-( o+i)]


=

-1.(!:_-1.)
3 6 3
n:-2
18

=--

(c)

HE6

3 rnarks

y=J;

Let

Correct solution.

:.x=y"

2 marks

dx = 2v
dy
.

Uses the given substitution correctly and


makes substantial progress to,vards a
correct solution.

dx=2ydy

I I
I.v~6-y')
I

1 mark

dx

~x(l-x) -

2ydy

Uses the given substitution correctly.

~/(!-/)

2vdv

dv

=2

J(1~y')

= 2sin- 1 y+ c
but

y=J;
2sin- 1 J;+c

d,

. . ~x(l-x)

(d)

HE2

n! > 2n, for all positive integral values of n ~ 4

Prove true for 11 = 4


LHS=4!

RHS= 2'

=24

=16

24 > 16

:. True for n = 4

3 marks
Correct solution.
2 marks
Prove the relationship is true for n=4 and
1nakes substantial progress to,vards a
correct solution.

1 mark
Correctly proves the relationship true for
n=4.

Assmne true for n = k

ie. Assun1e k!> 2t

k!-2! >0

Prove true for 11 = k + l


ic. Prove (k+1)!>2'''
Consider the difference

( k+ I)!- 2'" = ( k +I) .k!- 2.2'

=k.k!+k!-2-2 1
= kk!-2k +k1-2
= (k-1}.k!+ k!-2' +k!-2'
= ( k-1 }.k!+ 2( k!-2')
Now, since k > 4, (k-1) > 0

k!>O
:.(k-1).k!>O

Also, k!-2k > O. from the assmnption

Hence. (k-l}k!+2(k!-2')>0
(k+l)!-2''' >0
:.(k+1}!>2'''

..

By the process ofn1athematical induction,

11! > 2" is t1ue for all positive integral values of 11;?. 4

Section1(10marks)
Attemptquestions110.Usethemultiplechoiceanswersheetprovided.
1.

Evaluate lim
x 0

(A)

3 sin 7 x

5x

(B)

2.

21

5

(C)

(A)

(B)

(C)

(D)

Forwhatvaluesof x is

(D)

15

x4
6?
x 1

3.

Theintervaljoiningthepoints A 3,2 and B 9, y isdividedexternallyintheratio5:3bythe


point P x,13. Whatarethevaluesof x and y ?

(A)

x 27, y 22

(B)

x 18, y 4

(C)

x 6, y 12

(D)

x 27, y 4

4.

A circle with centre O has a tangent TU, diameter QT, STU = 25o and RPS= 22o .

What is the size of RTQ ?


(A)
(B)
(C)
(D)

22
25
43
47

5.

Forthepolynomialequation 6 4 x 10 x 2 8 x 3 0 ,thesumofitsroots,whendividedbythe
productofitsrootswouldbe:

(A)

(B)

(C)

(D)

JRAHS 2015 TRIAL EXT 1 MATHEMATICS

Page 1

6.

7.

Aparticlemovessuchthatwhenitisxmetresfromtheoriginitsaccelerationisgivenby
1
a e x .Whatisitsvelocitywhen x 3 ,giventhat v 1 when x 0 ?
2
(A)
0.050 ms 1 (B)
0.070 ms 1 (C)
0.158 ms 1 (D)
0.223 ms 1

Which of the following is the correct expression for

dx
36 x 2

x
c
6

(A) cos -1

(B) cos -1 6x c

(C) sin -1

(D) sin -1 6x c

8.

Eden,Tobyandfourfriendsarrangethemselvesatrandominacircle.Whatistheprobability
thatEdenandTobyarenottogether?

(A)

x
c
6

1

120

(B)

(C)

(D)

119

120

9.

If t tan

(A)

whichofthefollowingexpressionsisequivalentto 4 sin 3 cos 5 ?

2t 2

1 t2
2

(B)

t 42
1 t2

2t 2

1 t2
2

(C)

(D)

t 42
1 t2

10.

Anexpressionforthegeneralsolutiontothetrigonometricequation tan 3 x 3 where n is


anyintegeris:

(A)

n

3
9

(B)

n

3 3

(C)

n

3
3

(D)

n 2

3
9

JRAHS 2015 TRIAL EXT 1 MATHEMATICS

Page 2

SectionII(60marks)

Attemptallquestionsfrom1114.Answereachquestiononaseparatepage.

Question11(15marks)
(a)

Thenumberofanimalsinalocalfarmwhowillbeinfestedwithavirusadherestotheequation
p
where n =thenumberofanimalsinfestedbythevirus
n
1 Ce kt
p =thetotalnumberofanimals
k =thegrowthconstant
t =thetimeinmonths
C=constant

Thefarmernoticesthatinitially1animaloutoftheanimalpopulationof200isinfestedwiththe
virus.Afteronemonththenumberofanimalsinfestedwiththevirusincreasesto5.

(i)

(ii) Showthat k =1.63(to3significantfigures)

(iii) Howmanyanimalscanthefarmerexpecttobeinfestedafter3months.

(b)

(i)

(ii)

Showthatafter t months, n

Find

(d)

d 2x
x
tan 1

2
dx 4 x
2
2

Henceevaluate
0

(c)

200

1 199e kt

dx

4 x

2 2

Asphericalmetalballisbeingheatedsuchthatthevolumeincreasesatarateof
5 mm3 / min . Atwhatrateisthesurfaceareaincreasingwhentheradiusis3 mm .

Findanexpressionfor

e3 x
dx usingthesubstitution u 1 e x .
x
1 e

Question12(15marks)Startanewpage

(a)

Agroupof15studentsfromalocalschoolisselectedfortraininginsoccertorepresentthe
schoolatgradesport.Howeveronlyateamof11playersistobechosenfortheWednesday
game.
TheprobabilitythataplayerwillnotbeavailabletoplayonWednesdayduetoinjuryorother
commitmentsis0.14.

(i)

Findtheprobabilitythat3studentswillnotbeavailablefortheWednesdaygradesportin
soccer.Answerto3decimalplaces.

(ii)

Writethenumericalexpressionfortheprobabilitythattheteamwillbeunabletomakeup
ateamofallfit11players.Youdonothavetosimplifytheanswer.

JRAHS 2015 TRIAL EXT 1 MATHEMATICS

Page 3

(b)

Let P 2ap, ap 2 and Q 2aq, aq 2 betwopointsontheparabola x 2 4ay. Thesecant PQ passes


throughthepoint Aa,0 , andthetangentsat P and Q meetat R.

(i)

Showthat p q 2 pq.

(ii)

Findthecoordinatesof R intermsof p and q.

(iii) As P and Q vary,showthat R movesonastraightline.

(iv) Findtherestrictionsonthe x valuesofthelocusofR.

(c)

Usemathematicalinductiontoprovethatforallintegers n 3,

2
2
2 2 2
.
1 1 1 ...........1
n nn 1
3 4 5

Question13(15marks)Startanewpage

(a)
(i)Usingtheauxiliaryanglemethodexpress 3 sin 2t 2 cos 2t intheform r sin2t .

Aparticlemoveshorizontallyinastraightlinesothatitsposition x fromafixedpointattime t is
givenby:
x 3 sin 2t 2 cos 2t 2

Displacementismeasuredinmetresandtimeinhours.

(ii) Findanequationtorepresenttheaccelerationofthisparticleandprovethatitismovingin
simpleharmonicmotion.

(iii) Giventhattheparticleisattheoriginatnoon,betweenwhattimeswill
theparticlebemorethanonemetretotherightoftheoriginforthefirsttime(Letthetime
at t = 0 benoon).Giveyourtimescorrecttothenearestminute.

(b)

(c)

Considerthefunction y

(i)
(ii)

(iii)

Findthedomainandrangeofthefunction.
Sketchthegraphofthefunctionshowingclearlythecoordinatesoftheendpoints.

1
Theregioninthefirstquadrantboundedbythecurve y cos 1 x 1 andthecoordinate
2
axesisrotatedaboutthe y axis.Findthevolumeofthesolidofrevolution,givingyour
answerinsimplestexactform.

x
Whatistheexactvalueofthedefiniteintegral 3 sin 2 dx ?
4
0

JRAHS 2015 TRIAL EXT 1 MATHEMATICS

1
cos 1 x 1.
2

Page 4

2
1

Question14(15marks)Startanewpage
(a)

InaBMXdirtbikecompetitionthetakeoffpoint O foreachcompetitorwaslocatedatthetopof

thedownslope.Theanglebetweenthedownslopeandthehorizontalis . Thebikertakesoff
3

from O withvelocity V m / s atanangle tothehorizontal,where 0 . Thebikerlands


2
onthedownslopeatsomepoint Q, adistance D metresfrom O.

Theflightpathofthebikerisgivenby
1
x Vt cos and y gt 2 Vt sin
2
where t isthetimeinsecondsaftertakeoff.
(DONOTPROVETHIS)

(i)

(ii)

ShowthattheCartesianequationoftheflightpathofthebikerisgivenby
gx 2
y x tan 2 sec2 .
2V

Showthat
V2
D 4 cos 3 cos sin .
g

(iii) Showthat
dD
V2
4
cos 2 3 sin 2 .
d
g

(iv) Showthat D hasamaximumvalueandfindthevalueof forwhichthisoccurs.

(i) Consideringtheidentity 1 x n 1 x n 1 x 2 n , where n isapositiveinteger,

(b)

showthatforintegervaluesof r ,
2r

1
k 0

(ii)

k n

Ck nC2 r k 1 nCr provided0


r

Henceshowthat 1

Ck nC2 r k

k n

k 0

(iii)

Henceevaluate 1
k 0

12

1
1r nCr 1 nCr for0
2

Ck asabasicnumeral.

ENDOFTHEEXAMINATION
JRAHS 2015 TRIAL EXT 1 MATHEMATICS

Page 5

NORTHERN BEACHES SECONDARY COLLEGE

MANLY SELECTIVE CAMPUS


HIGHER SCHOOL CERTIFICATE
Trial Examination
2015

Mathematics Extension 1
General Instructions

Reading time 5 minutes


Working time 2 hours
Write using black or blue pen
Write your Student Number at the
top of each page
Answer Section I- Multiple Choice
on Answer Sheet provided
Answer Section II Free Response
in a separate booklet for each
question.
Board approved calculators and
templates may be used.

Section I Multiple Choice

10 marks
Attempt all questions

Section II Free Response

60 marks
Each question is of equal value
All necessary working should be shown
in every question.

Weighting: 40%

Manly Selective Campus


2015 HSC Mathematics Extension 1 Trial

Multiple Choice: Answer questions on provided answer sheet.


Q1.

The diagram shows a circle with centre O. The line PT is tangent to the
circle at the point T. TOP = 4x and TPO = x.

4x

T
What is the value of x?
(A)

(B)

18

(C)

36

(D)

72

Q2.

Which of the following is a simplified expression for

(A)

sin x

(B)

cos x

(C)

tan x

(D)

cot x

Q3.

The point P divides the interval AB in the ratio 3:7. In what external ratio
does the A divide the interval PB
(A)

3:10

(B)

3:4

(C)

7:3

(D)

10:3

Page 2 of 10

Manly Selective Campus


2015 HSC Mathematics Extension 1 Trial
Q4.

What is the obtuse angle between lines


(A)

15o

(B)

75o

(C)

105o

(D)

165o

Q5.

What is the value of

(A)
(B)
(C)
(D)

Q6.

In how many ways can 5 people be selected from a group of 6 and then
arranged in a line so that the two oldest people in the selected group are at
either end of the line? (NB. No two people are the same age.)
(A)

720

(B)

144

(C)

72

(D)

36

Q7.

The remainder of the division


(A)

(B)

(C)

x+2

(D)

x+1

is equal to

Page 3 of 10

Manly Selective Campus


2015 HSC Mathematics Extension 1 Trial

The power of x in the 7th term of the expansion of

Q8.
(A)

(B)

-3

(C)

(D)

-5

Q9.

is

The velocity of a particle is given by the equation


is initially located at the origin, what displacement at t = 3?
(A)

(B)

(C)

15

(D)

16

Q10.

The diagram show the graph of a cubic function

Which is a possible equation of this function?


(A)
(B)
(C)
(D)
End of Multiple Choice
Page 4 of 10

. If the particle

Manly Selective Campus


2015 HSC Mathematics Extension 1 Trial

Question 11: Start A New Booklet

(a)

Evaluate

(b)

(i)

Verify that (++ ) = ++ + 2(++).

(ii)

Hence, or otherwise, if , and are the roots of

(c)

(d)

15 Marks

(3)

(1)
,

evaluate

(3)

(i)

Determine the vertical asymptotes for

(2)

(ii)

Hence sketch the curve

(2)

Find the general solution of the equation


(4)

Page 5 of 10

Manly Selective Campus


2015 HSC Mathematics Extension 1 Trial

Question 12 Start A New Booklet

(a)

Use the substitution

(b)

A particle moves with acceleration

15 Marks

to show

(3)

. Initially, the particle is one metre

to the right of the origin and its velocity is 4m/s.


Find the displacement of the particle when it is at rest.

(c)

(3)

ABCD is a cyclic quadrilateral. The tangents from Q touch the circle at A and B.
The diagonal DB is parallel to the tangent AQ, and QA produced intersects CD
produced at P.
Let < QAB = .

C
NOT TO
SCALE
D

(d)

(i)

Prove that BAD is isosceles, giving reasons.

(2)

(ii)

Find < DCB in terms of , stating reasons.

(1)

(iii)

Show that P,C, B and Q are concyclic points

(2)

Show that x = 1.8 is a reasonable approximation for the x value of the


point of intersection of y = 2sin x and y = x in the
domain 0.78 < x < 2.35.

(1)

Use Newtons method with one application to find a better


approximation to the x value of this point on intersection.

(3)

(i)

(ii)

Page 6 of 10

Manly Selective Campus


2015 HSC Mathematics Extension 1 Trial

Question 13 Start A New Booklet


(a)

15 marks

The diagram shows the parabola

The tangent to the parabola at


P cuts the y-axis at N.

cuts the x-axis at T and the normal at

The equation to the tangent is given by


(i)

Show the coordinates of N are

(2)

(ii)

Let M be the midpoint of NT. Find the Cartesian equation of the locus.

(3)

Use mathematical induction to prove that ( n + 1) + n 1 is divisible by 2


2

(b)

for all integers n 1 .

(c)

(3)

A school band is to be formed with a brass section containing 8 students and a


percussion section containing 4 students.
(i)

(ii)

In how many ways can the band be formed if 12 students audition for the
brass section and 10 students audition for the percussion section?

(1)

In how many ways can the band be formed if it is certain that Maria will be
successful for the brass audition and Marcus will be successful for the
percussion audition?

(2)

Question 13 continues on next page

Page 7 of 10

Manly Selective Campus


2015 HSC Mathematics Extension 1 Trial

Question 13 continued
(d)

At time t years the number N of individuals is given by

for some

constants a > 0 , b > 0. The initial population size is 20 and the limiting population
size is 100.

(i)

Show that

(2)

(ii)

Find the values of a and b.

(2)

Page 8 of 10

Manly Selective Campus


2015 HSC Mathematics Extension 1 Trial

Question 14

(a)

15 marks

Warehouse A has 100 computers and the probability that of selecting a


computer which is defective is 0.02.
Warehouse B has 100 computers, two of which only, are defective.
Joe buys three computers from Warehouse A and three computers from
Warehouse B. What is the probability that exactly one of the computers he has
bought is defective?

(b)

(3)

Two towers T1 and T2 have heights h metres and 2h metres respectively. The
second tower is due south of the first tower. The bearing of tower T1 from a
surveyor is 292. The bearing of the tower T2 from the surveyor is 232. The
angle of elevation from the surveyor to the top of tower T1 is 30 while the
angle of elevation from the surveyor to the top of tower T2 is 60.

Show that the distance d between the two towers is given by d =

Question 14 continues on next page

Page 9 of 10

21h
3 metres.

(3)

Manly Selective Campus


2015 HSC Mathematics Extension 1 Trial

Question 14 continued.
(c)

A ball is projected vertically from the ground with a speed of 49 m/s. The height y of
4.9t 2 + 49t. (Do NOT show this.)
the ball at time t is given by y =
At the same time, a second ball is projected from the ground into the air with an
angle of projection . Its horizontal displacement is given by x = 98t cos and its
4.9t 2 + 98t sin . (Do NOT show this.)
height is given by y =
(i)

Find the maximum height of the ball that was projected vertically.

(2)

(ii)

Find the value of at which the second ball should be projected if it is to hit
the first ball when the first ball reaches its maximum height.

(2)

Find the horizontal distance between the two balls when they are first
projected into the air. Give your answer in exact form.

(1)

(iii)

(e)

Consider the binomial expansion

Show that, if n is even:

(4)

End of Examination

Page 10 of 10

2015 MSC HSC X1 Trial Solutions

Q1

Q2

Q3

Q4

Page 1 of 14

2015 MSC HSC X1 Trial Solutions

Q5

Q6

Q7

Q8

Page 2 of 14

2015 MSC HSC X1 Trial Solutions

Q9

Q10

Cubic of form

Page 3 of 14

2015 MSC HSC X1 Trial Solutions


Q11

3 marks
correct solution

2 marks
- correct
integrand

1 mark
simplification to

1 mark
expansion
demonstrated

Page 4 of 14

2015 MSC HSC X1 Trial Solutions

3 marks
correct solution
2 marks
correct
rearrangement
of initial term
1 mark
correct
values for

2 marks
correct answer
1 mark
equating
denominator to
zero

2 marks
- shape
- x-intercept
-

Page 5 of 14

2015 MSC HSC X1 Trial Solutions

4 marks
- correct solution
3 marks
-correct solution for
without stating
reason for excluding
cot
2 marks fully
factorised and use of
trig identity
1 mark first
factorisation

Page 6 of 14

2015 MSC HSC X1 Trial Solutions


Q12

3 marks correct solution


2 marks final integral
simplified to cos2
1 mark correct initial
substitution including
limits and d

Page 7 of 14

2015 MSC HSC X1 Trial Solutions

3 marks correct
solution
2 marks attaining
constant correctly
1 mark use of

2 correct solution
1 mark one
correct use of
geometrical
principle.

c-i

c-ii

1 correct solution

Page 8 of 14

2015 MSC HSC X1 Trial Solutions

2 marks
correct solution
c-iii

1 mark
determination
of size of

1mark correct
solution

d-i
The solution is close to zero therefore reasonable approximation.

3 marks
correct solution
2 marks
correct
expression for
xa

d-ii

1 mark
incorrect
substitution into
initial formula

Page 9 of 14

2015 MSC HSC X1 Trial Solutions

Q13
2 marks-correct solution
showing all steps
1 mark- correct equation
of normal

a)i)

3 marks- correct
equation showing all
steps
2 marks-correct values
for midpoint
1 mark- correct x value
for T

ii)

Page 10 of 14

2015 MSC HSC X1 Trial Solutions


3 marks- correct solution
showing all steps
2 marks-partial correct
with insertion of
assumption into S(k+1)

b)

1 mark- correct for n=1

1 mark- correct solution

c)i)

2 marks- correct solution

ii)

1 mark- correct
selections for
brass/percussion only
2 marks correct
solution
1 mark- correct initial
expression for dN/dt

d)i)

Page 11 of 14

2015 MSC HSC X1 Trial Solutions


2 marks- correct solution
1 mark- only one correct
value

ii)

Question 14
P(exactly one computer defective)
=P(1 defective from A, 0 from B) +P(0 from A, 1 from B)

3 marks: correct solution


2 marks: obtaining a
correct binomial prob.
1 mark: 1st line showing
understanding of options
required.

3 marks: correct solution


2 marks:correct
expressions for triangles,
attempting to use them in
cos rule
1 mark: correct
expressions for triangles

Page 12 of 14

2015 MSC HSC X1 Trial Solutions

2 marks: correct solution


1 mark: finding
solving

and

ii
2 marks: correct solution
1 mark: substituting t=5
and y=122.5 into2nd
equation

iii
1 mark: correct solution

Page 13 of 14

2015 MSC HSC X1 Trial Solutions


d

4 marks: correct solution


incl. explanation of
for n even
3 marks: correct solution
without justifying sign of
last term
2 marks: differentiating
twice and substituting
either x=1 or x=-1
1 mark: differentiating
twice

Page 14 of 14

BOSTES Number: ____________________________


CLASS (Please circle): 12M1 12M2 12M3 12M4 12M5

NORMANHURSTBOYSHIGHSCHOOL
N E W

S O U T H W A L E S

2015
H IGH ER SCHOOL C ERTIFICATE
T R IA L E X A M IN A T I O N

Mathematics Extension 1
General Instructions

Reading time - 5 minutes


Working time - 2 hours
Write using black or blue pen
Board-approved calculators may be
used
A table of standard integrals is provided
at the back of this paper
In Questions 11-14, show relevant
mathematical reasoning and/or
calculations
Begin each question on a separate
writing booklet

Total marks - 70
Section I

Pages 2-4

10 marks
Attempt Questions 1-10
Answer on the Multiple Choice answer sheet
provided
Allow about 15 minutes for this section
Section II

Pages 5-9

60 marks
Attempt Questions 11-14
Allow about 1 hour 45 minutes for this section

Students are advised that this is a school-based examination only and cannot in any way
guarantee the content or format of future Higher School Certificate Examinations.

Section I
10 marks
Attempt Questions 1 10
Allow about 15 minutes for this section
Use the multiple-choice answer sheet for Questions 1-10

1
y
NOT TO
SCALE

y = f '( x)

-4

-2

The diagram above represents a sketch of the gradient function of the curve y f ( x ) .
Which of the following is a true statement? The curve y f ( x ) has
(A)

a minimum turning point occurs at x 4

(B) a horizontal point of inflexion occurs at x 2


(C) a horizontal point of inflexion occurs at x 4
(D) a maximum turning point occurs at

Solve
(A)

3
1
x 2 x 1
x 1, 0 x

(B) 1 x 0 and

1
3
and x
2
2
1
3
x
2
2

(C)

3
1
x and 0 x 1
2
2

(D)

1
3
x , x 0 and x 1
2
2
2

0.

If

, evaluate

2 .

(A)
(B)
(C)

(D)

4 Find the acute angle between the tangents to the graphs y x and y x3 at the point (1,1).

(A) 27
(B) 30
(C) 45
(D) 63
5

The polynomial
Find the value of k.

2 has

(A) 7
(B) 7
(C) 12
(D) 12
6

Find
(A)

(B)

sin

2
1 x2
1

1 x2
(C) cos 1 x
(D) sin 1 x
3

2 as a factor.

Find the domain and range of


3 cos
3 Range: 0
(A) Domain: 0
(B) Domain: 1
(C) Domain:
(D) Domain:

1 Range: 0

Range: 0

Range: 0

A stone is thrown at an angle of to the horizontal. The position of the stone at time t
1
seconds is given by x Vt cos and y Vt sin gt 2 where / is the
2
acceleration due to gravity and v m/s is the initial velocity of projection.
What is the maximum height reached by the stone?
V sin
(A)
g
(B)

g sin
V

V 2 sin 2
(C)
2g
g sin 2
2V 2
The volume of a sphere of radius 8 mm is increasing at a constant rate of 50 mm 3 .
(D)

Determine the rate of increase of the surface area of the sphere.


(A) 0.06 mm2 /s
(B) 0.60 mm2 /s
(C) 1.25 mm2 /s
(D) 12.50 mm2 /s
10 The acceleration of a particle is defined in terms of its position by
2
particle is initially 2 to the right of the origin, travelling with velocity 6
minimum speed of the particle.
(A) 6

(B) 16

(C) 20
(D) 36

4 . The
. Find the

Section II
60 marks
Attempt Questions 11 14
Allow about 1 hour and 45 minutes for this section

Answer each question in a SEPARATE writing booklet.


Your responses should include relevant mathematical reasoning and/or calculations.

Question 11 (15 marks)

Use a SEPARATE writing booklet.

(a)

(b)

Marks

(i)

Evaluate

(ii)

Use Simpsons rule with 3 function values to approximate

(iii)

Use your results to parts (i) and (ii) to obtain an approximation for e .
Give your answer correct to 3 decimal places.

Using one application of Newtons method with = as the first


2

1
2

approximation, find the second approximation to the root of the equation


2
3. Correct answer to 3 decimal places.

(c)

3
2
The polynomial P( x) x bx cx d has roots 0, 3 and 3.

(i)
(ii)

What are the values of b, c and d?


Without using calculus, sketch the graph of y P ( x ) .

(iii)

Hence or otherwise, solve the inequality

x2 9
0
x

(d)

tan
. Find the slope of the tangent to the
Consider the function

curve where the function y f ( x ) cuts the y-axis.

(e)

Find the exact value of

4 x2 dx , using the substitution x 2sin .

2
1
1

Question 12 (15 marks)

Use a SEPARATE writing booklet.

Marks

(a)

28,19 divides the interval AB externally in the ratio : 5 .


The point
Find the value of k if A is the point 4,3 and B is the point 2, 1

(b)

(i)

sin x cos x
Show that sin( x )
4
2

(ii)

Hence or otherwise, solve

(c)
(i)

and

sin x cos x
3

for 0 x 2 .
2
2

2
are two points on the parabola x 4ay .

Show that the equation of the normal to the parabola at P is given by

x py 2ap ap3 .
(ii)

Find the co-ordinates of R, the point of intersection of the normals at


P and Q, in terms of p and q.

(iii)

If

(d) Evaluate

(e)

2, find the cartesian equation of the locus of R.

using the substitution

After t years the number of animals, N, in a national park decreases


according to the equation:
dN
0.09( N 100)
dt
The initial number of animals in the national park is 400.
(i)

Verify that N 100 Ae 0.09 t is a solution of the above equation,


where A is a constant.

(ii)

How many years does it take for the number of animals to reach 150?

Question 13 (15 marks)

Use a SEPARATE writing booklet.

Marks

(a)

The diagram shows a cylindrical barrel of length l and radius r. The point A
is at one end of the barrel, at the very bottom of the rim. The point B is at the
very top of the barrel, half-way along its length. The length of AB is d.

(b)

(i)

Show that the volume of the barrel is

(ii)

Find l in terms of d if the barrel has maximum volume for the given d.

1
2

Two circles are intersecting at P and Q. The diameter of one of the circles is
PR.

Copy this diagram into your writing booklet.


(i)
(ii)

Draw a straight line through P, parallel to QR to meet the circle PRQ at


S and the other circle at T. Prove that QS is also diameter of the circle.
Prove that the circles have equal radii if TQ is parallel to PR.

Question 13 continues on page 9


7

2
2

Question 13 (continued)
(c)

A rocket is fired from a pontoon on the sea. The rocket is aimed at a 60m
high cliff, 240m from the pontoon. The angle of projection of the rocket is
45 and its initial velocity is 402
.
(i)

Taking the point of projection as the origin O, derive expressions for


the horizontal component x and vertical component y of the position
of the rocket at time t seconds.
(Assume the acceleration due to gravity is 10

(ii)

Show that the path of the rocket is given by the equation

(iii)

Find the time taken for the rocket to land on top of the cliff.

(iv)

Find the exact velocity of the rocket when it reaches this point.
(Hint: velocity includes magnitude and direction)

Question 14 (15 marks)

Use a SEPARATE writing booklet.

(a)

Use mathematical induction to prove that 3


1.
for all integers

(b)

The velocity of a particle moving in a straight line is given by


10
where x metres is the displacement from a fixed point O and v is the velocity
in metres per second. Initially the particle is at O.

(c)

is divisible by 4

(i)

Show that the acceleration of the particle is given by


10

(ii)

Express x in terms of time t.

(iii)

What is the limiting position of the particle?

Marks
3

1
3
1

A particle moves in a straight line and its displacement x metres from a fixed
point O at any time t seconds is given by the equation
x 4 cos 2 t 1 .
(i)

Prove that the particle is undergoing simple harmonic motion.

(ii)

State the period of the motion.

(iii)

Sketch the graph x 4 cos 2 t 1 for 0 t .


Clearly show the times when the particle passes through O.

(iv)

Find the time when the velocity of the particle is increasing most
rapidly for 0 t .

End of paper

STANDARD INTEGRALS
n

x dx

1 n+1
x , n -1; x 0, if n < 0
n 1

x dx

ln x, x > 0

dx

1 ax
e , a0
a

cos ax dx

1
sin ax, a 0
a

sin ax dx

1
- cos ax, a 0
a

sec

ax dx

1
tan ax, a 0
a

sec ax tan ax dx

1
sec ax, a 0
a

1
dx
x2

1 -1 x
tan , a 0
a
a

ax

1
a2 x2
1
x a
2

1
x a
2

dx

x
sin -1 , a 0, -a < x < a
a

dx

ln x x 2 a 2 , x a 0

dx

ln x x 2 a 2

NOTE: ln x loge x, x 0

10

2015 HSC ASSESSMENT TASK 3

Mathematics
Extension 1
General Instructions
Class Teacher:

Reading time 5 minutes


Working time 2 hours
Write on one side of the paper
(with lines) in the booklet provided
Write using blue or black pen
Board approved calculators may
be used
All necessary working should be
shown in every question
Each new question is to be started
on a new page.
Attempt all questions

(Please tick or highlight)

Mr Berry
Mr Ireland
Mr Lin
Mr Weiss
Ms Ziaziaris
Mr Zuber

Student Number:_________________________
(To be used by the exam markers only.)
Question
No

Mark

1-10

11

12

13

14

Total

Total

10

15

15

15

15

70

100

Section I
10 marks
Attempt Questions
Allow about 15 minutes for this section
Use the multiple choice answer sheet for Questions

_____________________________________________________
1.

What is the value of

(A) 0
(B)
(C) 1
(D)

2.

is a linear function with gradient , find the gradient of


(A) 4
(B)
(C)
(D)

3.

Which of the following best describes the above function?


(A)
(B)
(C)
(D)

4.

What are the coordinates of the point that divides the interval joining the points A(
B(
externally in the ratio 1:3?
(A)
(B)
(C)
(D)

5.

Which of the following is the solution to


(A)
(B)
(C)
(D)

and

6.

The polynomial

has roots ,

and . What is the value of ?

(A) 2
(B)
(C) 4
(D)

7.

The line TA is a tangent to the circle at A and TB is a secant meeting the circle at B and C.

Given that TA = 4, CB = 6 and TC = x , what is the value of x?


(A) 2
(B) 4
(C) 6
(D)8

8.

Given that

, find an expression for

(A) 2
(B) 4
(C) 8
(D) 16

9.

Find the gradient of the normal to the parabola


(A)
(B)
(C)
(D)

at the point where

10.

An approximate solution to the equation


is
Newtons method, a more accurate approximation is given by:
(A)

(B)

(C)

(D)

. Using one application of

Section II
60 Marks
Attempt Questions
Allow about 1 hour and 45 minutes for this section
Answer each question on a NEW page. Extra writing booklets are available.
In Questions

, your responses should include relevant mathematical reasoning and/or calculations.

______________________________________________________________________
Question 11 (15 Marks) Start a NEW page.
(a)

When the polynomial


is the value of a?

is divided by

the remainder is 7. What


2

(b) (i)
1

(ii)
1

(iii)
2
(iv)
2
(c)

Find the acute angle between the lines

(d)

Evaluate

and

(e)

Find the general solution to

Question 12 (15 Marks) Start a NEW page.

(a) (i)

(ii)

(b)

Without using calculus, sketch the graph of

Hence solve

Using the substitution

find the exact value of:

(c) (i)

A chef takes an onion tart out of the fridge at


into a room where the air temperature is
. The rate at which the onion tart warms follows Newtons law, that is:

where k is a positive value, time t is measured in minutes and temperature T is measured in


degrees Celsius.
Show that

is a solution to

and find the value of A.

(ii)

The temperature of the onion tart reaches

in 45 minutes. Find the exact value of k.

(iii)

Find the temperature of the onion tart 90 minutes after being removed from the fridge.

Question 12 continues on page 8

(d) (i)

ABC is a triangle inscribed in a circle. MAN is the tangent at A to the circle ABC.
CD and BE are altitudes of the triangle.
Copy the diagram into your answer booklet.

(ii)

Give a reason why BCED is a cyclic quadrilateral

(iii)

Hence show that DE is parallel to MAN.

End of Question 12

Question 13 (15 Marks) Start a NEW page


(a)

Is the graph of

identical to

? Give a reason for your answer.

(b) (i)

A particle is moving in a straight line. At time t seconds it has displacement x metres from a
fixed point O on the line, velocity
and acceleration
given by
. Initially
the particle is 5m to the right of O and moving towards O with a speed of 6
.
Explain whether the particle is initially speeding up or slowing down.

(ii)

Find an expression for

(iii)

Find where the particle changes direction.

Express

(c) (i)

(ii)

in terms of .

in the form

Hence, or otherwise, solve

for

(d) (i)

A square ABCD of side 1 unit is gradually pushed over to become a rhombus. The angle at A
decreases at a constant rate of 0.1 radian per second.
At what rate is the area of rhombus ABCD decreasing when

?
3

(ii)

At what rate is the shorter diagonal of the rhombus ABCD decreasing when
3

Question 14 (15 Marks) Start a NEW page.


(a)

Prove that

is a multiple of 10 for all positive integers

(b) (i)

Show that

(ii)

Hence, using a similar expression, find a primitive for

(iii)

The curves

and

The curve

also intersects with the x axis at Q.

intersect at

Find the area enclosed by the x-axis and the arcs OP and PQ.

(c) (i)

A parabola has parametric equations

Sketch the parabola showing its orientation and vertex.

(ii)

Point

is the point on the parabola where

Point

is the point on the parabola where

Find the equation of the locus of the midpoint of

(iii)

A line with gradient m passes through

and state its geometrical significance

and cuts the parabola at distinct points Q and R.

Find the range of possible values for m.

End of Examination.

NORTH SYDNEY GIRLS HIGH SCHOOL

2015 TRIAL HSC EXAMINATION

Mathematics Extension 1
General Instructions

Total marks 70

Reading Time 5 minutes

Section I

Working Time 2 hours

10 marks

Write using black or blue pen


Black pen is preferred
Board approved calculators may be used
A table of standard integrals is provided at
the back of this paper

Attempt Questions 1 10
Allow about 15 minutes for this section

In Questions 11 14, show relevant


mathematical reasoning and/or calculations

Attempt Questions 11 14
Allow about 1 hour and 45 minutes for this
section

NAME:______________________________

Section II

110

Pages 7 14

60 Marks

TEACHER:___________________

STUDENT NUMBER:__________________________

QUESTION

Pages 2 - 6

MARK

11

/10
total/15

12

/15

13

/15

14

/15

TOTAL

/70

Section I
10 marks
Attempt Questions 110
Allow about 15 minutes for this section
Use the multiple-choice answer sheet for Questions 110.

2 x3 3x 2 5 x 7
x
4 x x3

What is the value of lim

(A)

1
2

(B)

(C)

(D)

1
2

Which of the following is equivalent to

(A)

2sin
6

(B)

2sin
6

(C)

2sin
3

(D)

2sin
3

3 sin cos ?

A curve is defined by x 2t and y log e t .


Which of the following is the value of

(A)

1
4

(B)

1
2

(C)

(D)

What is the value of lim


x 0

(A)

2
3

(B)

3
2

(C)

4
9

(D)

( )

dy
at the point 2, 0 ?
dx

2sin 2 x
?
3 tan 3x

What is the value of sin , given that ACD in the diagram below?

(A)

3 5

2
3

(B)

2
5

5 3

(C)

2 2

5 3

(D)

4
3 5

1
3

dx
?
What is the correct expression for
4 x2

(A)

x
sin 1 c
4

(B)

x
sin 1 c
2

(C)

1 1 x
sin c
4
4

(D)

1 1 x
sin c
2
2

The graph below represents the depth of water in a channel (in metres) as it changes
over time (in hours).

Which of the following is NOT true?

(A)

The centre of motion is at 8 m

(B)

The period of oscillation is 8 hours

(C)

The amplitude is 8 m

(D)

The rate of change in the depth of water is the fastest when the depth is 8 m

Which of the following are the roots of the equation x3 4 x 2 x 6 0 ?


(A)

1, 3, 2

(B)

1, 3, 2

(C)

1, 1, 6

(D)

1, 1, 6

10

What is the value of cos 1 sin where 2 ?


(A)

(B)

(C)

(D)

x 1
2
within the natural domain, three students obtain the

x 1
x
following inequalities.
In solving

Student I:

x 1

Student II:

x 1

Student III:

x 1

2 x

2 x 1 x

x 2 x x 1

Which students will obtain the correct solution to the original inequality?

(A)

Student I only

(B)

Student II only

(C)

Student III only

(D)

Student II and Student III

Section II
Total marks 60
Attempt Questions 1114
Allow about 1 hour 45 minutes for this section.
Answer each question in a SEPARATE writing booklet. Extra writing booklets are available.
In Questions 11 to 14, your responses should include relevant mathematical reasoning and/or
calculations.

Question 11 (15 marks) Use a SEPARATE writing booklet.

(a)

Differentiate x cos1 ex with respect to x.

(b)

Find sin 2 3x dx

(c)

The point P divides the interval joining A -1, 5 to B 2, 3 externally


in the ratio 4 : 3 . Find the coordinates of P.

(d)

Find the size of the acute angle between the line y 2 x and the curve

( )

y x 2 at the point of intersection 2, 4 .


Give your answer to the nearest degree.

(e)

dx

Use the substitution u x to determine


.
1 1 x x

Give your answer in exact form.

(f)

(i)
(ii)

x
Sketch the graph of y sin
for the domain 3 x 3 .
2
Hence, or otherwise, find for what positive values of m, the equation
x
sin
mx , has exactly three solutions.
2
7

1
2

Question 12 (15 marks) Use a SEPARATE writing booklet.

(a)

Angela is preparing food for her baby and needs to use cooled boiled water.
The equation y = Aekt describes how the water cools, where t is the time in
minutes, A and k are constants and y is the difference between the water
temperature and the room temperature at time t, both measured in degrees
Celsius.
The temperature of the water when it boils is 100C and the room temperature
is a constant 23C.
(i)

Find the value of A.

(ii)

The water cools to 88C after 5 minutes. Find the value of k correct to
three significant figures.

(iii) Angela can prepare the food when the water has cooled to 50C.
How much longer must she wait?

(b)

A particles displacement satisfies the equation t x 2 5 x 4 , where x is


measured in cm and t is in seconds. Initially, the particle is 4 cm to the
right of the origin.

1
.
2x 5

(i)

Show that the velocity is given by v

(ii)

Find an expression for the acceleration, a in terms of x.

(iii) Find the position of the particle 10 seconds after the start of the motion.

(iv)

Briefly describe the motion of the particle.

Question 12 continues on page 9

Question 12 (continued)

(c)

BAC and BAD are two circles such that the tangents at C and D meet at T
on AB produced.

B
D
T

Copy or trace the diagram into your writing booklet.

If CBD is a straight line prove that:


(i)

TCAD is a cyclic quadrilateral

(ii)

TC = TD

End of Question 12

Question 13 (15 marks) Use a SEPARATE writing booklet.

(a)

()

(i)

Fully factorise P x = x 3 - x 2 - 8x +12 .

(ii)

Hence, find any values of k, such that Q x 0 for all real x,

()

( )(

()

where Q x = P x 2x - k .

(b)

In the diagram below, OL is a road that runs due east. OM is another road and
intersects OL at 30. Both roads are on flat ground. On OM there are three
equally spaced vertical telegraph poles AB, CD and EF of equal height h m.
The distance between adjacent poles is twice the height of the poles.
From an observer at P, the bearing of the first pole AB is 300T. The angles of
elevation of A and C from P are 45 and 30 respectively.

(i)

Explain why triangle BDP is right angled.

(ii)

Deduce that PF h 13 .

Question 13 continues on page 11

10

Question 13 (continued)

(c)

Consider the parabola x 2 4 y .


P 2 p, p 2 and Q 2q, q 2 lie on the parabola.

Q 2q, q 2
P 2 p, p 2

(i)

Find the equation of the chord PQ.

(ii)

Show that if PQ is a focal chord then pq 1 .

(iii) T 2t,t 2 , t 0 and R 2r,r 2 are two other points on the parabola
distinct from P and Q.
If TR is also a focal chord and P, T, Q and R are concyclic,
show that p 2 + q 2 = t 2 + r 2 .

End of Question 13

11

Question 14 (15 marks) Use a SEPARATE writing booklet.

(a)

A particle is undergoing simple harmonic motion such that its


displacement x centimetres from the origin after t seconds is given by :

x 2 4sin 2t .
3

(b)

(i)

Between which two positions is the particle oscillating?

(ii)

At what time does the particle first move through the origin in the
positive direction?

Use the principle of mathematical induction to prove 3n 7 4n for all

integers n 3 .

Question 14 continues on page 13

12

Question 14 (continued)

(c)

Consider the region enclosed by the circle x a y 2 a 2 and the line x b


2

shown in the diagram below, where 0 b 2a .


y

x
b

(i)

Show that the volume of the spherical cap formed by rotating this
region around the x-axis is given by

(ii)

b2
3

3a b

cubic units

A spherical goldfish bowl of radius 10 cm is being filled with water


at a constant rate of 75 cm3 per minute.
Using part (i) or otherwise, find the rate at which the water level in
the bowl is rising when the bowl is half full of water.

Question 14 continues on page 14

13

Question 14 (continued)

(d)

Consider the function f x x

(i)

By restricting the domain of the original function to x 0 ,


find the equation of f

(ii)

1
whose graph is shown below.
x

x .

Hence, without solving directly, find the value(s) of x


for which

x 1
16 . Leave your answer in exact form.
x

No marks will be awarded for solving the equation directly for x.

End of paper

14

Mathematics Extension 1 Trial HSC 2015 Suggested Solutions


Section I
1.
D
Degree of numerator and denominator is the same. The
limit is the ratio of the leading coefficients ie

2
2 .
1

2.
B
Using auxiliary angle method, this is o the form

R sin where R

3 (1) 2 2 .

B using standard integrals table

7.
C
The amplitude is the distance from the centre of motion
to the extreme of motion which is 12 8 = 4.
8.
B
Sum of roots = 4 and product of roots is 6.
9.

tan
.
6
3

3.

6.

cos

sin cos 1 sin ;


cos 1 cos 2 ;

dy dy dx

.
dx dt dt
dy 1 dx
dy 1
dy 1
;
2;
. At 2, 0 , t 1; .
dt t dt
dx 2t
dx 2

2sin 2 x 2
sin 2 x
lim
lim
x 0 3 tan 3 x
3 x 0 tan 3 x
2 2
sin 2 x
3x
lim
lim
x 0 tan 3 x
3 3 x 0 2 x
4

9
5.

ACB

sin sin BCD

acute

cos 1 cos 2 ; 2 acute

Use parametric differentiation.

4.

acute

2
5 1 2 2
2
.

.
5 3
5 3 3 3 5

using compound angle result

Alternately sub a second quadrant angle into your


calculator and verify which option works.
10.

As

x 0 it is not necessary to multiply by the square

of

x only by the square of x 1 as Student II has

done. However by multiplying by the square of x


Student III does not generate extra solutions because
x 0 is not an admissible solution.

Mathematics Extension 1 Trial HSC 2015 Suggested Solutions


Section II
Question 11
(a)
Using chain rule,

d
1
x cos 1 ex 1.cos 1 ex x
e

2
dx
1 ex
cos 1 ex
(b)

1 e2 x 2

Using double angle results,

sin

(c)

ex

1 cos 6 x
3 xdx
dx
2

1
sin 6 x
= x
c
2
6

Using a ratio of 4 : 3

4 2 3 1 4 3 3 2
P
,
11, 3
4 3
4 3

(d)

y 2 x; m1 2 and y x 2 ; y ' 2 x; m2 4 at x 2
tan

m1 m2
24 2

1 m1m2
1 8 9

13 (nearest degree)
(e)

du
1
dx

2du
dx 2 x
x
1
1
x u
; x 1 u 1
3
3

u x

dx

1
1 x x
3

1
2.du

2 tan 1 u 1
2
1 1 u
3
3


=2
4 6 6

(f)

(i)
(ii)

The upper bound of m to ensure exactly three solutions is


found by finding the gradient of the tangent at x 0 and
is

. As we need positive values of m, then the

required range of values for m is 0 m

Question 12
(a)

(i)

At t 0; y 100 23 77

77 Ae0 A 77

(ii)

t 5; T 88 23 65
65 77e5 k
65
77
65
5k ln
77
1 65
k ln 0.0339 (4dp)
5 77
e5 k

(iii)

50 23 77ekt
27
e kt
77
27
kt ln
77
1 27
t ln
k 77
t 30.928 30m 55s

Therefore, she must wait another 25 min 55 sec.

(b)

(i)

(ii)

t x2 5x 4
dt
2x 5
dx
dx
1
v

dt 2 x 5

d 1 2
v
dx 2

d 1
1
a

dx 2 2 x 5 2
1
2

2
2 2 x 5 3
a

2 x 5

(iii)

When t 10

10 x 2 5 x 4
x2 5x 6 0

x 6 x 1 0
x 1, 6
Initially, x 4 so v

1
1
0
2(4) 5 3

So the particle is moving to the right. v can never be zero so the particle never turns around. So it can never be
at x 1 . x 6 .
(iv)

(c)

Initially the particle is 4 units to the right and moving to the right. the acceleration at this time is
negative, so the particle is slowing down.

(i)
Let TCB and TDB

TCB CAB (Angle between tangent and chord equal to angle in alternate segment)
TDB DAB (Angle between tangent and chord equal to angle in alternate segment)
CAD CAB DAB (1)
Now, CTD 180 (Angle sum of CTD ) (2)

CAD CTD 180 (adding (1) and (2))


TCAD is a cyclic quadrilateral. (opposite angles are supplementary)

(ii)

CAT CDT (angles in the same segment in circle TCAD)



This means that TCB TDB
TC TD (equal sides opposite equal angles in TCD )

Question 13
(a)

(i)

P 2 0 x 2 is a factor

P x x 2 x 2 x 6 by inspection. [Alternately use long division].


P x x 2 x 3 x 2
P x x 2 x 3
2

(ii)

Q x P x 2 x k

Q x x 2 x 3 2 x k
2

k
2

2 x 2 x 3 x
2

If Q x 0 for all x, then x x 3


2

Or k 6

(b)

(i)

BP h cot 45 h and

DP h cot 30 h 3
BP 2 DP 2 h 2 3h 2 4h 2
2h BD 2
2

BDP is right angled at P (converse of Pythagoras Theorem)

3h
h 60

In BPF , BF 2h 2h 4h
(ii) DBP tan 1

Using the cosine rule,

PF 2 BP 2 BD 2 2.BP.BD.cos PBF
h 2 4h 2 .h.4h.
2

1
2

17 h 2 4h 2 13h 2
PF 13h as required

(c)

q p q p p q
q2 p2

2q 2 p
2
2 q p

(i) mPQ

Equation of PQ is:

y p2

pq
x 2 p
2

2 y 2 p 2 p q x 2 p 2 2 pq
y

pq
x pq
2

(ii) Focus is 0,1 . Sub into eqn of chord PQ

1 0 pq

or

pq 1

(iii) PQ and TR are chords of circle PTQR and intersect at the focus S.

PS SQ TS SR
(product of intercepts of intersecting chords)
But PS p 2 1 using the locus definition; distance from focus = distance from directrix
Similarly, QS q 2 1 etc

p 2 1 q 2 1 t 2 1 r 2 1
p2q2 p2 q2 1 t 2r 2 t 2 r 2 1
2 2
But pq 1 p 2 q 2 1 and tr 1 t r 1

p2 q2 t 2 r 2

Question 14
(a)

(i)

(ii)

Centre of motion is 2 . Amplitude is 4. Hence, oscillates between 6 and + 2.


Solving for x 0

4sin 2t 2
3

sin 2t
3 2

5 13
2t , ,
,...
3 6 6 6
11
2t , ,
,... as t 0
6 2 6
11
t ,
,...
4 12
Graph of displacement is as below:

Hence, crosses the origin in a positive direction the second time ie at t


crosses in a positive direction.

(b)

To prove 3n 7 4n for n 3

Test if true for n = 3:


LHS = 33 7 27 7 34 and RHS = 43 64
LHS < RHS, hence true for n 3 .
Assume the result is true for some n = k where k 1; k

ie assume that 3k 7 4k
Prove true for n = k + 1 ie prove that 3k 1 7 4k 1

3k 7 4k by assumption. Multiply both sides by 3.


3k 1 21 3.4k
3k 1 7 14 3.4k
3k 1 7 3.4k
3k 1 7 4.4k
3k 1 7 4k 1
Hence, the proposition is true for all n 3 by Mathematical Induction.

11
. Alternately, use v 0 to find when it
12

(c)

(i) y 2 a 2 x a

V y 2 dx
0

a 2 x a dx
b

x 2 2ax a 2 dx
b

x3
ax 2

2

2 0
3
b 3


ab 2 0

b3 3ab 2

3
3
b2
V
3a b
3
(ii) a 10 and

b2

dV
75
dt

30 b 10 b2

b3

3
3
dV

20 b 3 b 2
db
3
dV dV db
(Chain Rule)

dt
db dt
db dV dV

dt
dt db
db
75 20 b b 2
dt
When the bowl is half full, b 10
db
75
3
75 200 100

dt
100 4
(d)

(i) y x

1
x

For the inverse: x y

1
y

Multiply by y

xy y 2 1

y 2 xy 1 0
x x2 4
2
As y 0 for the inverse, then
y

x x2 4
2

(ii)

x 1
1
16 x
16
x
x

x 16

x f 1 16

x 8 65

16 162 4
8 65
2

129 16 65
End of solutions

PENRITH HIGH SCHOOL


2015
HSC TRIAL EXAMINATION

Mathematics Extension 1

General Instructions:

Reading time 5 minutes


Working time 2 hours
Write using black or blue pen
Black pen is preferred
Board-approved calculators may
be used
A table of standard integrals is
provided at the back of this paper
In questions 11 14, show
relevant mathematical reasoning
and/or calculations
Answer each question on a new
sheet of paper

Student Number:

Total marks70
SECTION I

Pages 35

10 marks
Attempt Questions 110
Allow about 15 minutes for this section
SECTION II

Pages 69

60 marks
Attempt Questions 1114
Allow about 1 hours 45 minutes for this section

Teacher Name:

This paper MUST NOT be removed from the examination room

Assessor: T Bales
1

Section I
10 marks
Attempt Questions 110
Allow about 15 minutes for this section

Use the provided multiplechoice answer sheet for Questions 110


1

For > 1, ln is:


(A)
(B)
(C)
(D)

=0
>0
<0

Consider the function () given in the graph below:


f(x)

Which domain of the function f(x) above is valid for the inverse function to exist?:
(A)

> 0

(C)

0 < <

(B)

(D)
3

< 0

What is the acute angle between the lines 2 7 = 0 and 3 5 2 = 0 ?


(A)

(B)
(C)
(D)
3

< <

4 24

32 28

57 32

85 36

A particle is moving in a straight line with velocity / and acceleration / 2 . Initially the particle
started moving to the left of a fixed point . The particle is noticed to be slowing down during the course of
the motion from 0 to . It turns around at , keeps speeding up for the rest of the course of motion, passing
and and continues. The particle never comes back. Take left to be the negative direction.
A

During the course of the particles motion from to , which statement of the following is correct?

(A)

> 0 and > 0

(C)

< 0 and > 0

(B)

(D)

> 0 and < 0

< 0 and < 0

A particle is moving in a straight line with velocity,

1+

where is the displacement of the

particle from a fixed point . If the particle was observed to have reached the position = 2 at a certain
moment of time, then this particle:
(A)
(B)
(C)
(D)

will definitely reach the position = 1


may reach the position = 1

will never reach the position = 1

will come to rest before reaching = 1

If the rate of change of a function = () at any point is proportional to the value of the function at that
point then the function = () is a:

(A)

Polynomial function

(B)

Trigonometric function

(C)

Exponential function

(D)

Quadratic function

Let and be any two acute angles such that < . Which of the following statements is correct ?
(A)
(B)
(C)
(D)

<

<

<
<

Which of the following is a primitive function of 2 + 2 ?

(B)

(D)

2 + 2 +

1
4

1 + 2 2 + + = 21

1 + 2 + + = 21

(B)
(C)

1 + 2 2 + + = 2+1

(D)

1 + 2 + + = 2+1

Using the substitution, = 1 + , find the value of

(A)
(B)
(C)

(D)

3
3

2 +

Consider the binomial expansion (1 + ) = 1 + 1 +2 2 + + .


Which of the following expressions is correct?
(A)

10

1
2

1
4

3
3

2 + 2 +

(C)

1
2

2 +

(A)

6
5

1
3
2
3

3
2

1+

is:

Section II
60 marks
Attempt Questions 1114
Allow about 1 hour and 45 minutes for this section
Begin each question on a new sheet of paper. Extra sheets of paper are available.
In questions 1114, your responses should include relevant mathematical reasoning and/or calculations

Question 11 (15 marks) Start a new sheet of paper.


(a)

( + 1) and ( 2) are factors of () = 3 4 2 + + 6. Find the third factor.

(b)

Find the coordinates of the point which divides the interval internally in the ratio 2: 3
with (3, 7) and (15, 6)

(c)

Solve the inequality

(d)

Use the method of mathematical induction to prove that, for all positive integers :

12 + 32 + 52 + 72 + + (2 1)2 = (2 1)(2 + 1)

41

< 2,

graphing your solution on a number line.

(e)

B
Q
A

PT is the common tangent to the two circles which touch at T.


PA is the tangent to the smaller circle at Q, intersecting the larger circle at points and as shown.
i)

ii)

State the property which would be used to explain why 2 =


If = , = = , prove that m =

Proceed to next page for question (12)


6

nr
nr

Question 12 (15 marks) Start a new sheet of paper.


(a)

The equation = 1 2 has a root near = 0.3. Use one application of Newtons
methods to find a better approximation, giving your answer correct to 2 decimal places.

(b)

Five couples sit at a round table. How many different seating arrangements are possible if:

i)

there are no restrictions?

ii)

each person sits next to their partner?

6
5

(c)

In the expansion of (4 + 2 3 2 ) 2 , find the coefficient of 5

(d)

i)

Write the binomial expansion for (1 + )

ii)

Using part (i), show that

iii)

Hence show that

n
n
1 n
1
x
dx
Ck 3k +1
+
=

0 ( )
k =0 k + 1

n
=
Ck 3k +1

1
k
+
k =0

Proceed to next page for question (13)


7

1 n +1
4 1
n +1

Question 13 (15 marks) Start a new sheet of paper.


(a)

Use the substitution =

12

(You can use the result that

(b)

Give the exact value for

(c)

to show that
1

2 + 2

1 12 =

1 2

= 1 + . You do not need to prove this).

dx
3 + x2
3

The distinct points P, Q have parameters = 1 and = 2 respectively on the parabola


= 2, = 2 . The equations of the tangents to the parabola at P and Q respectively are
given by:
1 + 1 2 = 0 and 2 + 2 2 = 0 (You do not need to prove these)

ii)

Show that the equation of the chord 2 (1 + 2 ) + 21 2 = 0

) Prove that for any value of 1 , except 1 = 0, there are exactly two values
of 2 for which M lies on the parabola 2 = 4.

i)

iii)

Show that M, the point of intersection of the tangents to the parabola at P and Q,
has coordinates 1 + 2 , 1 2 .

Find these two values of 2 in terms of 1 .

Proceed to next page for question (14)


8

Question 14 (15 marks) Start a new sheet of paper.


(a)

A particle is moving in simple harmonic motion on the axis, according to the law
= 43 where is the displacement of in centimetres from at time seconds.

i)

State the period and amplitude of the motion.

ii)

Find the first time when the particle is 2cm to the positive side of the origin and
its velocity at this time.

iii)

Find the greatest speed and greatest acceleration of

(b)

Q
A projectile is fired from , with speed 1, at an angle of elevation of to the horizontal.
After seconds, its horizontal and vertical displacements from (as shown) are metres and metres,
repectively.
i)
ii)
iii)

1
2

Prove that = and = 2 +


Show that the time taken to reach is given by

The projectile falls to , where its angle of depression from is .


Prove that, in its flight from to , is the half-way point in terms of time.

End of paper

3
2
3

STANDARD INTEGRALS

x dx
e

1
x n +1 , n 1; x 0 , if n < 0
n+1

dx =

ax

= ln x , x > 0

dx =

1 ax
e , a0
a
1
sin ax , a 0
a

cos ax dx

sin ax dx

sec

ax dx =

sec ax tan ax dx
1

a2 + x2

dx =

1
a2 x2
1
x a
2

1
x +a
2

1
cos ax , a 0
a
1
tan ax , a 0
a
=

1
sec ax , a 0
a

1
x
tan 1 , a 0
a
a

dx = sin 1

x
, a 0, a < x < a
a

dx = ln( x + x 2 a 2 ), x > a > 0

dx = ln( x + x 2 + a 2 )

NOTE: ln x = loge x, x > 0

10

Student Number:_________________

11

Teacher Name:_____________________

St George Girls High School


Trial Higher School Certificate Examination

2015

Mathematics
Extension 1
General Instructions

Total Marks - 70

Section I - Pages 2 - 5
10marks
Attempt Questions 1-10.
Allow about 15 minutes for this section.
Answer on the sheet provided.

Reading time - 5 minutes


Working time - 2 hours
Write using blue or black pen.
Write your student number on each booklet.
Board-approved calculators may be used.
A table of standard integrals is provided at
the back of this paper.
The mark allocated for each question is
listed at the side of the question.
Marks may be deducted for careless or
poorly presented work.

Section II - Pages 6 - 11
60marks
Attempt Questions 11- 14.
Allow about 1 hour 45 minutes for this
section.
Begin each question in a new booklet.
Show all necessary working in
Questions 11- 14.

Students are advised that this is a Trial Examination only and does not necessarily
reflect the content or format of the Higher School Certificate Examination.

St George Girls High School


Trial HSC Examination - Mathematics Extension 1 - 2015

Page2

Section I
10marks
Attempt Questions 1 - 10
Allow about 15 minutes for this section
Use the multiple-choice answer sheet for Questions 1-10.
1.

In the diagram, AB is a diameter of the circle and MN is tangent to the


circle at C. LCAB = 35. What is the size of LMCA?
M

(A) 35

(B) 45

(C) 55

(D) 65

2.

Which function is graphed below?


I

--+------~-----+---

------""i------~----I

'

(A) 2ir sin 3x


1

''

'
'

'
'
--+------~-----+--'
'
''

TI

'
'
------""i------~
'

(B) Zn sin- 1 - x
'

'
'

(C) 4 sin- 1 3x

-----...f

(D) 4 sin- 1

_,

.,

'

'

--T-----

-1

''

I ----- 1
' ---

'

'
'
------~-----+---

'

------ ,------1
'
'

'
'

- - - - - 1'

'

''

~ ------ ,------,' -----''

3x

St George Girls High School


Trial HSC Examination - Mathematics Extension 1- 2015

Page3

Section I (cont'd)

3.

4.

Find 1- 1 (x), given f(x)

3x-3
=x-2

3y-3

(A)

f- 1 (x) = -

(B)

f- 1 (x) = -x-3

(C)

f- 1 (x)

= 3x-3

(D)

f- 1 (x)

3-3x
= -

x-2

Zx-3

x-2

2-x

Which diagram best represents y = -x(Z - x) 3 (x

(A)

(B)

,1

(C)

(D)

+ 1) 2 ?

St George Girls High School

Page4

Trial HSC Examination - Mathematics Extension 1 - 2015

Section I (cont'd)
5.

Find k given x - 2 is a factor of P(x)

=x3 -

3x 2

+ kx + 12

= -4

(A) k

(B) k = 0
(C) k = 2

=4

(D) k

6.

The acute angle between 11 : 2x - y - 3 and 12 : y = 3x + 7 is closest to:


(A) 15
(B) go
(C) 82

(D) 45

7.

J2cos x dx
2

+x +C
sin 2x + x + C

(A) sin x cos x

(B)

-i
2

(C) -cos 3 x
3

(D)

8.

-2

Y1-x 2

+C
+c

The velocity of a particle at a position x is

x=

2e-, metres per second.

Calculate the particle's acceleration when its displacement is -2 metres.


(A) -e m/s 2
4 I z
(B) --ms
eZ

(C) -2e 2 m/s 2


(D)

e 2 m/s 2

St George Girls High School


Trial HSC Examination - Mathematics Extension 1- Z015

Page 5

Section I (cont'd)
9.

Find the exact value of sin 15


(A)

(B)
(C)

(D)

3..fj,
2-.,fi.
2.,fi.

2(-../6 - ~)
../6 -.,fi.
4

10. Given the curve below, Eden intends to use Newtons Method to find an
approximation to the root shown. Which initial estimate will not produce a
good approximation with this method?

(A)

Xo

(B)

Xo

(C)

Xo

(D)

Xo

End of Section I

=a
=b
=C
=d

St George Girls High School


Trial HSC Examination - Mathematics Extension 1 - 2015

Page 6

Section II
60marks
Attempt Questions 11 - 14
Allow about 1 hours 45 minutes for this section
Answer each question in a SEPARATE writing booklet. Extra writing booklets are
available.
In Questions 11 - 14, your responses should include relevant mathematical
reasoning and/or calculations.
Question 11 (15 marks) Use a SEPARATE writing booklet
1

> -21

a)

Solve the inequality - 1x-1 1

b)

Sketch the intersection of y >

c)

Given A(-2, 3) and B(lO, 11), find the coordinates of the point P which
divides the interval AB in the ratio 3: 1.

lxl -

1 and y

<1

d)

You are given 3.6 as an approximate root of the equation x 3 - 50.


Use one application of Newton's method to find a better approximation.
(to 2 decimal places)

e)

If y = sin(ln x), find

(i)

dy
dx

Marks

(ii)

d2y
dx 2

1,2

St George Girls High School


Trial HSC Examination - Mathematics Extension 1- 2015

Page7

Marks

Question 11 (continued)
BC is tangent to the circle at B. Find the value of

f)

x, giving reasons.

St George Girls High School

Trial HSC Examination - Mathematics Extension 1- 2015

Page 8

Marks

Question 12 (15 marks) Use a SEPARATE writing booklet


a)

Solve 3 sinx

+ 4cosx =

2.5, 0:::::; x:::::; 2rr

b)

..e=...~~~~~~~~~~~..,._~~~___;;:::....~...;;;;,,=-~~----lQ
A

The tangents from Q touch the circle at A and B. PC and PQ are straight
lines LBAQ = a.
(i)

Copy or trace the diagram into your writing booklet.

(ii) Given PD = 5 cm and DC = 7 cm, calculate the exact length of AP.

(iii) Show that LBCD = 2a.

(iv) Show that PQBC is a cyclic quadrilateral.

St George Girls High School


Trial HSC Examination - Mathematics Extension 1 - 2015

Page 9

Marks

Question 12 (continued)

c)

The rate at which a body warms in air is proportional to the difference


between its temperature T and the constant temperature A of the
surrounding air. This rate can be expressed by the differential equation
dT

- = k(T-A)

dt

where t is time in minutes and k is constant.


Show that T =A+ Cekt where C is a constant, is a solution of the
differential equation.

(ii) A glass of milk warms from 4C to 8C in 15 minutes. The air


temperature is 25C. Find the temperature of the glass of milk after a
further 45 minutes, correct to the nearest degree.

(iii) With reference to the equation for T, explain the behaviour of T as t


becomes very large.

(i)

St George Girls High School


Trial HSC Examination - Mathematics Extension 1- 2015

Page 10

Marks

Question 13 (15 marks) Use a SEPARATE writing booklet


a)

Evaluate

b)

(i)

fo

x 3 (-Jx4 +

1) dx

using the substitution u = x 4

+ 1.

By considering its second derivative, show that y = ex - 4x is always


concave up.

(ii) Use the trapezoidal rule with 3 function values to find an approximation

h
5

to

(e:L- 4x) dx,

correct to 4 significant figures.

(iii) Is this approximation too large or too small? Justify your answer?
c)

SOm

x
200m---------'>

A projectile is launched from the top of a 50 m high building with an initial


speed of 40 m/s. It is launched at an angle of a 0 above the horizontal, as in
the diagram. Acceleration due to gravity is 10 m/s2.
(i)

d 2x
d 2y
Given that - =
0
and
- = -10, show that x = 40t cos a and
dt 2
dt 2

y =-5t 2 + 40t sin a

+ 50 where x and y

are the horizontal and vertical

displacements of the projectile in metres from O at time t seconds


after launching.

(ii) The projectile lands on the ground 200 metres from the base of the
building. Find the two possible values of a. Give your answers to the
nearest degree.

St George Girls High School


Trial HSC Examination - Mathematics Extension 1- 2015

Page 11

Marks

Question 14 (15 marks) Use a SEPARATE writing booklet


a)

b)

c)

P(2ap, ap 2 ) and Q(2aq, aq 2 ) are points on the parabola x 2


If p + q = 4, find the locus of M, the mid point of PQ.

= 4ay.

Given that x is a positive integer, prove by the method of mathematical


induction that (1 + x)n - 1 is divisible by x for all positive integers n 2:: 1.

The velocity v ms- 1 of a particle moving on a horizontal line is given by


v 2 = 252
(i)

+ 216x -

36x 2

Show that the particle is performing simple harmonic motion.

(ii) Find the centre of the motion.

(ii) Find the amplitude of the motion.

(iv) Find the period of the motion.

(v) Find the maximum speed of the particle.

(vi) Initially the particle is at one of the extreme points of the motion.
Where will it be when t = Brr seconds.

12

(vii) Find its average speed during the first

1
:;

End of Examination

seconds.

~OLUTIONS

;a ,s-

. ~-,-

HSC

,. =. s .',...
- I

-\

:>(..

~ ~ { J

-Jr L.
v -

~
I

-~~x.~1

<.!!
-

-i

Jj

-J

S~

11

zr

~ '1 ~ ... );r

-'

.!
3

.,,('

'1

- 4
_,

'1 -....
~

~,,..

- II
-<

2.

')(

-D.

~1-

.,!

3. ')(. -3.
~
i.

')(-

~,.... t,i10,-

,J

'>l

")(.

":...

3'-7 - ~
I

; ... ')_,

1 - ) x ~ 1 ',_ - 3
("K -3.) :. 2-~-3
,

-:.

..

:x.-3

So Le

TrctJ

~l

,<. =- 0

.._J-.

")<. :. -

.,._J..

')A-

ALL

A,

--

SJ

~(1)

,,.Q

d- 3

3.(~)'l- + k(2-)

I;..

-tJ.k.

1-

l'.l

::<'.)

=O

+12.

.)}.__

-f
;. -1-

")

"" :. .2
I

(''-,_ : 3
j

')

1 ) eas i:
=::.

"-

el1t

jL 1 ( ,

-f <-DS .,'}~)

( I
X-

+
+

c..,_s

2')\. )

l. .s ,---...

J d~
J '}(.

.l n. -+- c.

...

J3, - I
:)_ J2---

Ji_
')(

J'i-

~_Ji

'+

'""'--o
""" -- a.

SYDNEY BOYS HIGH SCHOOL


MOORE PARK, SURRY HILLS

2015
HIGHER SCHOOL CERTIFICATE
TRIAL PAPER

Mathematics
General Instructions

Extension 1
Total Marks 70

Reading time 5 minutes.

Working time 2 hours.

10 Marks

Write using black or blue pen.

Attempt Questions 110

Board approved calculators may be used.

All necessary working should be shown in

Allow about 15 minutes for this


section.

Pages 24

every question if full marks are to be awarded.

Section II

Marks may NOT be awarded for messy or

60 marks

badly arranged work.

Attempt Questions 1114

Leave your answers in the simplest exact form,

Allow about 1 hour and 45


minutes for this section

unless otherwise stated.

Section I

Start each NEW question in a separate answer


booklet.

Examiners:

Pages 611

R. Elliot & J. Chen

This is an assessment task only and does not necessarily reflect the content
or format of the Higher School Certificate.

Section I
10 marks
Attempt Questions 110
Allow about 15 minutes for this section
Use the multiple-choice answer sheet for Questions 110.
1

The roots of 3x3 2x2 + x 1 = 0 are , and .


What is the value of 2 + 2 + 2 ?
(A)
(B)
(C)
(D)

1
9
2

9
1
2
9

What is the minimum value of


(A)

(B)

(B)

16
7 3

(D)
3

7 sin x 3cos x ?

2x
What is the domain and range of y = sin 1 ?
5
(A)
Domain: 1 x 1 ;
Range: y
(B)

Domain: 1 x 1 ;

Range:

y
2
2

(C)

5
5
Domain: x ;
2
2

Range:

y
2
2

(D)

5
5
Domain: x ;
2
2

Range: y

Evaluate lim
x0

(A)
(B)
(C)
(D)

sin3x
.
2x

0
2
3
1
3
2
2

In the diagram below, AB is a tangent to the circle BCD.


Also, CD is a tangent to the circle ABD.
BAD = and BCD = .
NOT TO SCALE
B

C
A
D
Which of the following is a true statement?

(A)

ABD BDC

(B)

ABCD is a cyclic quadrilateral

(C)

ABD ||| BDC

(D)

AB || CD

A particle moves in simple harmonic motion so that its velocity, v, is given by


v2 = 6 x x2 .

Between which two points does it oscillate?

(A)

x = 6 and x = 3

(B)

x = 2 and x = 3

(C)

x = 1 and x = 2

(D)

x = 2 and x = 3

Which of the following is an expression for cos3 x sin x dx ?

4
(A)
cos x + c

(B)

1
cos 4 x + c
4

(C)

cos 4 x + c

(D)

1
cos 4 x + c
4

Which of the following is the correct expression for the inverse of f (x) = e12 x ?
(A)
(B)
(C)
(D)

10

f 1 (x) = 2e12 x
1
f 1 (x) = e12 x
2
1
f 1 (x) = log e (1 2x)
2
1
f 1 (x) = (1 log e x)
2

Three Mathematics study guides, four Mathematics textbooks and five exercise books are
randomly placed along a bookshelf. What is the probability that the Mathematics textbooks are
all next to each other?

(A)

4!
12!

(B)

9!
12!

(C)

4!3!5!
12!

(D)

4!9!
12!

A particle moves on the x-axis with velocity v m/s, such that v2 = 16x x2.
Which of the following is the particles maximum speed and the position of where
this maximum speed occurs?
(A)

Maximum speed = 16 m/s at x = 0

(B)

Maximum speed = 8 m/s at x = 8

(C)

Maximum speed = 8 m/s at x = 8

(D)

Maximum speed = 8 m/s at x = 8

Section II
60 marks
Attempt Questions 1114
Allow about 1 hour and 45 minutes for this section
Answer each question in a NEW writing booklet. Extra pages are available
In Questions 1114, your responses should include relevant mathematical reasoning and/or
calculations.
Question 11 (15 Marks)

Start a NEW Writing Booklet

(a)

Differentiate sin 1 (log e x) .

(b)

Find

(c)

(i)

Simplify sin ( A + B ) + sin ( A B ) .

(ii)

6
Hence, evaluate sin3x cos x dx.
0

1
4 9x 2

dx .

(d)

(e)

The point P 6 p, 3p 2 is a point on the parabola x 2 = 12 y .


(i)

Find the equation of the tangent at P.

(ii)

The tangent at P cuts the y-axis at B.


The point A divides PB internally in the ratio 1 : 2.
Find the locus of the point A as P varies.

A piece of meat at temperature T C is placed in an oven, which has a constant


temperature of H C.
The rate at which the temperature of the meat warms is given by
dT
= K (T H ) ,
dt
where t is in minutes and for some positive constant K.

(i)

Show that T = H + Be Kt , where B is a constant, is a solution of the


differential equation above.

(ii)

If the meat warms from 10 C to 50 C in the oven, which has a constant


temperature of 180 C, in 30 minutes, find the value of K.

(iii)

How long will it take the meat to get to a temperature of 150 C?


Express your answer correct to the nearest minute.

Question 12 (15 Marks)


(a)

(b)

Start a NEW Writing Booklet

(i)

Solve cos x 3sin x = 1 for 0 x 2 .

(ii)

Hence, or otherwise, find a general solution to cos x 3sin x = 1 .

(i)

On the same set of axes sketch the graphs of y = cos 2x and y =

(ii)

Use the graph to determine the number of solutions to the equation

x +1
3

3cos 2x = x + 1

(iii)

One solution of the equation 3cos 2x = x + 1 is close to 0.5.


Use one application of Newtons Method to find another approximation,
correct to 3 decimal places.

(c)

(d)

4
Evaluate sin 2 2x dx
0

When x cm from the origin, the acceleration of a particle moving in a straight


line is given by:

d 2x
5
=
2
3
dt
( x + 2)
It has an initial velocity of 2 cm/s at x = 0. If the velocity is V cm/s, find V in terms of x.

Question 13 (15 Marks)


(a)

Start a NEW Writing Booklet

In the diagram below, DC is a diameter of the larger circle centred at A.


AC is a diameter of the smaller circle centred at B.
DE is tangent to the smaller circle at F and DC = 12.

Copy the diagram to your answer booklet.


Determine the length of DE.

(b)

(i)

Simplify k! + k k!

(ii)

Prove, by mathematical induction, that

1 1! + 2 2! + 3 3! + ...+ n n! = (n + 1)! 1

for all positive integers n.

(c)

(i)

Using the substitution x = 3+ 3sin find

(ii)

Let R be the region bounded by the curve y = 4 x ( 6 x ) and the x-axis.

x ( 6 x ) dx

Find the volume of the solid of revolution generated by revolving R about


the x-axis.

End of Question 13

15

Question 14 (15 Marks)

SPECMATH EXAM 1 PT1

Start a NEW Writing Booklet

n 18
(a)
8 cm

1.2 m

10
cm
4m
m
hhcm

The figure above shows an inverted conical cup with base radius 8 cm
and height 10 cm.
rted cone, as shown in the diagram, is initially full of water. The water 2flows
out through a hole at the

Some
cm3 per minute.
3 water is poured into the cup at a constant rate of
at the rate of 0.1 h m per hour, where h m is the depth of water remaining
after t hours. The volume
5
3
water is given by VLet
= 0.03
h . of the water be h cm at time t minutes.
the depth

t hours,

dh
Find
is given
bythe rate of change in the area of the water surface when h = 4
dt

.9h 2

(b)

9h

3
2

.009h 2

A particle is projected horizontally at 30 ms 1 from the top of a 100 m high wall.


Assume that acceleration due to gravity is 10 ms 2 and that there is no air resistance.
The flight path of the particle is given by:
x = 30t, y = 100 5t 2 (Do NOT prove this)

1
3

0.9h 2
1

9h

3
2

where t is the time in seconds after take-off.


(i)

Find the time taken for the particle to reach the ground.

(ii)
Find the angle and speed at which the particle strikes the ground.
n 19
le starts from rest and moves in a straight line with acceleration 6 sin(2t) m/s2 at time t s.
acement from its starting point, in metres, Question
at time t is
by on page 11
14given
continues
4 sin(2t)
sin(2t) 3t
3 cos(2t)
5 sin(2t)
1.5 sin(2t)
10

1
2

Question 14 (continued)
(c)

The diagram below shows a tetrahedron such that VA = VB = AB = 2a,


CA = CB = 3a and VC = 5a .
O is the foot of the perpendicular from V to the base ABC.
M is the midpoint of AB.
P is a point on BC such that BP = ra where 0 r 3.
VMC = and VPO = .
V

5a
2a

3a

6
.
4

(i)

By considering VMC, show that cos =

(ii)

Hence find the exact value of VO.

(iii)

Show that VP 2 =

(iv)

Hence show that sin =

(v)

Hence, or otherwise, find the maximum value of as r varies.

1 2
3r 8r + 12 a 2
3

45

8 3r 8r + 12
2

End of paper

11

SYDNEY BOYS HIGH SCHOOL


M O O R E PA R K , S U R RY H I L L S

2015
HIGHER SCHOOL CERTIFICATE
TRIAL PAPER

Mathematics

Extension 1

Sample Solutions
Question Teacher
Q11
RB
Q12
BK
Q13
BD
Q14
PB

MC Answers
Q1
Q2
Q3
Q4
Q5
Q6
Q7
Q8
Q9
Q10

D
B
C
D
C
D
B
D
D
D

Section I

10 marks

The roots of 3x3 2x2 + x 1 = 0 are , and .


What is the value of 2 2 2 ?
(A)
(B)
(C)
(D)

1
9
2

9
1
2
9

ANSWER: D
3x 3 2x 2 x 1 0
d
b


a
a
1
2

3
3

2 2 2
1 2

3 3
2

What is the minimum value of


(A)

(B)

7 sin x 3cos x ?

ANSWER: B
(B)

16

7 sin x 3cos x
(D)

7 3

32

79
4
Let

7 sin x 3cos x r sin


4sin

No matter what the value of


1 sin x 1

4 4sin x 4
Therefore the minimum value is x 4.

(A)

Domain: 1 x 1 ;

2x
?
5
Range: y

(B)

Domain: 1 x 1 ;

Range:

(C)

Domain:

5
5
x ;
2
2

Range:

(D)

Domain:

5
5
x ;
2
2

Range: y

What is the domain and range of y sin 1

y
y

ANSWER: C

2x
2x
y sin 1 sin y
5
5

Domain: 1 sin y 1
2x
1
5
5
5
x
2
2

Range: y as this is the range of


2
2
1
y sin x
1

sin3x
.
x0
2x

Evaluate lim
(A)
(B)
(C)
(D)

0
2
3
1
3
2

ANSWER: D

lim
x 0

sin 3x 3
sin 3x 2
lim

2x
2 x 0 2 x
3
3
2sin 3x
lim
2 x 0 2 3 x
3
1
2
3

In the diagram below, AB is a tangent to the circle BCD.


Also, CD is a tangent to the circle ABD.
BAD = and BCD =
NOT TO SCALE
B

C
A
D
Which of the following is a true statement?
(A)

ABD BDC

(B)

ABCD is a cyclic quadrilateral

ANSWER: C

(C)

ABD ||| BDC

(D)

AB || CD

In ABD and DCB :


DCB DBA (angle in the alternate
segment)
ie DCB ABD
BAD BDC (angle in the alternate
segment)
ie BAD CDB
BD is but not respective to angles.
Therefore ABD DCB
Hence, triangles are equiangular they are
similar and ABD ||| DCB

A particle moves in simple harmonic motion so that its velocity, v, is given by

v2 6 x x2 .
Between which two points does it oscillate?
(A)

x = 6 and x = 3

(B)

x = 2 and x = 3

(C)

x = 1 and x = 2

(D)

x = 2 and x = 3

ANSWER: D

v2 6 x x2
For the particle to reach its oscillation
points v 0.
v2 6 x x2
0 6 x x2
0 (3 x)(2 x)
x 3 and 2
4

Which of the following is an expression for cos 3 x sin x dx ?

(A)

cos 4 x c

(B)

1
cos 4 x c
4

(C)

cos 4 x c

(D)

1
cos 4 x c
4

ANSWER: B

3
cos x sin x dx , testing solutions:

d
cos 4 x 4 cos 3 x sin x

dx
1
d
cos 4 x cos 3 x sin x
dx 4
1

cos 4 x cos 3 x sin x dx


4

Which of the following is the correct expression for the inverse of f (x) e12 x ?
(A)
(B)
(C)
(D)

f 1 (x) 2e12x
1
f 1 (x) e12 x
2
1
f 1 (x) log e (1 2x)
2
1
f 1 (x) (1 log e x)
2

ANSWER: D

Let y e1 2 x

y e12 x
ln y 1 2 x
2 x 1 ln y
1
1 ln y
2
1
f 1 ( x) 1 ln y
2
x

Three Mathematics study guides, four Mathematics textbooks and five exercise books are
randomly placed along a bookshelf. What is the probability that the Mathematics textbooks are
all next to each other?

(A)

4!
12!

ANSWER: D

(B)

9!
12!

(C)

4!3!5!
12!

(D)

Since there are 9 elements counting the


textbooks as 1 element, hence these can
be arranged in 9! ways. Also the
textbooks can be arranged in 4! ways.
As there are 12 separate elements, the
divisor for population can be counted in
12! ways.

4!9!
12!

Therefore, the probability is

10

9!4!
12!

A particle moves on the x-axis with velocity v m/s, such that v2 = 16x x2.
Which of the following is the particles maximum speed and the position of where
this maximum speed occurs?
(A)

Maximum speed = 16 m/s at x = 0

(B)

Maximum speed = 8 m/s at x = 8

ANSWER: D

(C)

Maximum speed = 8 m/s at x = 8

(D)

Maximum speed = 8 m/s at x = 8

v 2 16 x x 2
x2
1 2
v 8x
2
2
2x
d 1 2
v 8
2
dx 2
8 x

x 8 x
x 0 the speed is the greatest so,
When

x 8 x
0 8 x
x 8
At x 8,
v 2 16 x x 2
16(8) 8

64
v 8
As v, velocity can take positive and
negative values, but the speed can only
be positive, the maximum speed is
8 m/s.
6

(i) Most students realised they needed the auxiliary angle


method. Common errors included:
-evaluating tan incorrectly and having 1/sqrt(3)
-not finding all the solutions in the given domain.
(ii) Some had the incorrect formula.

An inaccurate graph resulted in the wrong number of solutions.


Using Newton's Method done well on the whole. Some
students did not use the given starting value and so were
incorrect.

The most common mistake was not using the double


angle formula correctly.

The most common error was to differentiate the given function in


terms of t.
Half a mark was deducted if no statement about sign of v was
included.

SolutionsQ13X1THSC2015
Averagemark:11.31/15

0
6

0.5
18

1
138

Mean
0.91

0
7

0.5 1 1.5 2 2.5 3 3.5 4 Mean


6 11 14 25 8 20 22 49 2.70


0
3

0.5
5

1 1.5
11 4

2 2.5 3 3.5 4 Mean


20 15 25 24 55 2.93

0
2

0.5
8

1
2

1.5
3

2
0

2.5
4

3 Mean
143 2.77

0
15

0.5
10

1
21

1.5
4

2
24

2.5
23

3
65

Mean
2.05

Sydney Girls High School


2015
TRIAL HIGHER SCHOOL CERTIFICATE
EXAMINATION

Mathematics Extension 1
General Instructions

Reading Time 5 minutes


Working time 2 hours
Write using black or blue pen
Black pen is preferred
Board-approved calculators may
be used
A table of standard integrals is
provided at the back of this paper
In Questions 11 14, show
relevant mathematical reasoning
and/or calculations

Total marks 70
Section I

Pages 3 5

10 Marks
Attempt Questions 1 10
Answer on the Multiple Choice answer sheet provided
Allow about 15 minutes for this section
Section II

Pages 6 9

60 Marks
Attempt Questions 11 14
Answer on the blank paper provided
Begin a new page for each question
Allow about 1 hour and 45 minutes for this section

THIS IS A TRIAL PAPER ONLY


Name: ....

Teacher: ...

It does not necessarily reflect the format


or the content of the 2015 HSC
Examination Paper in this subject.

Section I
10 marks
Attempt Questions 110

Allow about 15 minutes for this section


Use the multiple-choice answer sheet for Questions 1-10.

1. A committee of six is to be formed from seven women and nine men. Find the number of
committees possible if exactly two members of the committee are to be men.
(A)

1260

(B)

2646

(C)

36 036

(D)

60 480

85 . What is the size of angle ACB ?


34 and ADE =
2. In the diagram below BAC =

(A)

51

(B)

56

(C)

61

(D)

60

ex
3. Use the substitution u = e x to determine which of the following is an expression for
dx .

1 + e2 x

(A)

tan 1 ( e x ) + C

(B)

tan 1 ( e 2 x ) + C

(C)

(D)

2 (1 + e x )

e x

(1 + e )

x 2

+C

+C

~3~

4. The radius of a spherical balloon is increasing at the rate of 2 cm/s. The rate at which the volume
of the balloon is increasing when the radius is 10 cm is :
(A)

200 cm3 / s

(B)

400 cm3 / s

(C)

800 cm3 / s

(D)

100 cm3 / s

5. A stone is thrown vertically upwards with a speed of 21 m/s. How long is the stone in the air
before it reaches its maximum height? (Assume acceleration due to gravity is 10 m/s2.)
(A)

4.2 s

(B)

0.48 s

(C)

0.95 s

(D)

2.1 s

6. The polynomial equation f ( x ) = x3 + x 1 has a root near x = 0.5 . Using this as the initial
approximation, determine another approximation (correct to four decimal places) to the root using
one application of Newtons method.
(A)

x = 0.7141

(B)

x = 0.7142

(C)

x = 0.7143

(D)

x = 0.7144

dy
a
7. If y = sin 1 , then
=
dx
x

(A)

(B)

(C)

(D)

a
x2 a2
a
x2 a2
a
x x2 a2
a
x x2 a2

~4~

20

8. What is the value of

k =1

(A)

1 048 574

(B)

1 048 575

(C)

1 048 576

(D)

1 048 577

20

Ck ?

9. Given that a , b and c are the roots of the equation x 3 3 x 2 + mx + 24 =


0 , and that a and b
2
are the roots of the equation x + nx 6 =
0 , then the value of n is :
(A)

(B)

(C)

(D)

10. The sum 14 + 24 + 34 + 44 + ... + n 4 is given by the expression


The value of a b is :

(A)

25

(B)

25

(C)

(D)

~5~

6n5 + an 4 + bn3 n
.
30

Section II
Total 60 marks
Attempt Questions 1114
Allow about 1 hour and 45 minutes for this section
Answer all questions, starting each question on a new page.
In Questions 1114, your responses should include relevant mathematical reasoning and/or calculations.

Question 11
(a) Solve

Marks

(15 marks)

2
3 .
x 5

x
(b) Sketch the function f ( x ) = cos 1 , clearly indicating the domain and range of
2

the function.
(c) The velocity of a particle when x m from the origin is given by=
v 2 x 2 e3 x + 4 .

Find the acceleration of the particle when x = 1 .

(d) Find the general solution to the equation

(e) Find the value of

3 tan + 1 =
0.

lim 5 x + sin 3 x
.
x0
2x

(f) In the diagram, AOB is the diameter of

the circle with centre at O . TC is a


tangent to the circle at the point C such
that AC bisects TAB . Copy the
diagram onto your writing paper.
Prove that AT is perpendicular to TC .

End of Question 11
~6~

Question 12

Marks

(15 marks)

(a) By considering the derivative of ln ( tan x ) , find cosec 2x dx .

10

2
(b) In the expansion of x + 2 , find the coefficient of x .
x

(c) P ( 4 p, 2 p 2 ) and Q ( 4q, 2q 2 ) are two variable points on the parabola x 2 = 8 y . The
tangents at P and Q intersect at the point T .
(i)

Derive the equation of the tangent at P .

(ii)

Hence, show that the point T has the coordinates ( 2 ( p + q ) , 2 pq ) .

(iii)

Given that p 2 + q 2 =,
10 determine the cartesian equation of the locus of T .

(d) Find

( sin

x + 2 cos 2 x + 3 tan 2 x ) dx .

1
(e) Prove that tan 1 ( x + 1) + cot =
x tan 1 ( x 2 x 1) for x > 0 .

End of Question 12

~7~

Question 13

Marks

(15 marks)

(a) The growth rate per month of the number N of birds on a property during a drought
is 20% of the excess of the bird population over 1000.
(i)

Express the information in the form of a differential equation and show that
=
N 1000 + Ae

0.2 t

(where t is the time in months) is a solution to this

differential equation.
(ii)

Given that initially there are 8000 birds on the property, find the amount of
time that will elapse before the population is reduced to half.

(b) Ansett Airlines offer two options on all flights for their meal service chicken or

beef (vegetarians choose not to fly with Ansett). If 60% of the time Ansett
passengers select the chicken dish, what is the probability that out of 7 randomly
selected passengers at least 2 will select chicken for their meal?

(c) An iPhone is thrown from the top of a building, 6 metres high, with an initial velocity
of 8 m/s at an angle of 30 to the horizon.
(i)

Using 10 m/s2 for acceleration due to gravity, derive the horizontal and

vertical equations of motion for the iPhone.


(ii)

Determine the greatest height of the iPhone above ground level.

(iii)

Find the velocity and direction of the iPhones path after 1 second.

(d) Prove the following statement is true by mathematical induction for all integers n 1 .
n

r ( r !=) ( n + 1)! 1

r =1

End of Question 13

~8~

2
2

Question 14

Marks

(15 marks)

(a) A particle moves in a straight line with simple harmonic motion. At time t seconds,

its displacement x metres from a fixed point O , is given by x =


2 + 5sin 3t + .
4

9 ( x 2 ) .
(i) Show that x =
(ii) Determine the maximum speed of the particle and its displacement at this
time.

(b) How many different arrangements of the word MAMMOTH can be made if only five

1
2

letters are used?


(c) Use the substitution u 2= x + 1 to find the volume of the solid formed by rotating the
x 1
area bounded by the curve y =
, the x axis and the lines x = 3 and x = 8
x +1
about the x axis. Express your answer in exact form.
(d) Use the expansion of (1 + x ) to prove that

n + ( 1)
( 1)
1n
1
=
C1 nC2 + +
n +1
2
3
n
n

3
n
n

Cn 1 .

(e) Given that f ( x ) = Ax3 + Bx 2 + Cx + D is a function with a double zero at x = 1 , and


with a minimum value of 4 when x = 1 , find the values of A, B, C and D .

End of paper

~9~

CANDIDATE NUMBER

SYDNEY GRAMMAR SCHOOL

2015 Trial Examination

FORM VI
MATHEMATICS EXTENSION 1
Wednesday 5th August 2015

General Instructions

Collection

Writing time 2 hours

Write your candidate number on each


answer booklet and on your multiple
choice answer sheet.
Hand in the booklets in a single wellordered pile.

Write using black or blue pen.

Board-approved calculators and templates may be used.


A list of standard integrals is provided
at the end of the examination paper.
Total 70 Marks
All questions may be attempted.
Section I 10 Marks
Questions 110 are of equal value.
Record your answers to the multiple
choice on the sheet provided.
Section II 60 Marks

Hand in a booklet for each question


in Section II, even if it has not been
attempted.
If you use a second booklet for a question, place it inside the first.
Write your candidate number on this
question paper and hand it in with your
answers.
Place everything inside the answer
booklet for Question Eleven.

Questions 1114 are of equal value.


All necessary working should be shown.
Start each question in a new booklet.

Checklist
SGS booklets 4 per boy

Multiple choice answer sheet


Candidature 112 boys

Examiner
PKH

SGS Trial 2015 . . . . . . . . . . . . . . Form VI Mathematics Extension 1 . . . . . . . . . . . . . . Page 2


SECTION I - Multiple Choice
Answers for this section should be recorded on the separate answer sheet
handed out with this examination paper.

QUESTION ONE
Which of the following is an odd function?
(A) f(x) = tan1 x
(B) f(x) = cos x
(C) f(x) = sin(x

)
4

(D) f(x) = cos1 x

QUESTION TWO
Suppose is the acute angle between the lines y 2x = 3 and 3y = x + 2. Which of the
following is the value of tan ?
(A) 7
(B) 7
(C) 1
(D) 1
QUESTION THREE

70
O

What is the size of 6 ABC?


(A) 110
(B) 145
(C) 140
(D) 130
Exam continues next page . . .

SGS Trial 2015 . . . . . . . . . . . . . . Form VI Mathematics Extension 1 . . . . . . . . . . . . . . Page 3


QUESTION FOUR
What is the inverse function of f(x) = x2 + 1 for x 0?

(A) f 1 (x) = x 1, for x 0

(B) f 1 (x) = x 1, for x 0

(C) f 1 (x) = x 1, for x 1

(D) f 1 (x) = x 1, for x 1


QUESTION FIVE
x2 + x 6
.
Find lim
x2
x2
(A)
(B)
(C) 5
(D) 5

QUESTION SIX

x
5
7

Find the length of x.

(A) 35

(B) 12

(C) 60

(D) 84

Exam continues overleaf . . .

SGS Trial 2015 . . . . . . . . . . . . . . Form VI Mathematics Extension 1 . . . . . . . . . . . . . . Page 4


QUESTION SEVEN
1
If f(x) = tan1 , find f 0 (x).
x
x2
(A)
1 + x2
(B)
(C)

1
1 + x2

1
1 x2

(D)

x2
1 x2

QUESTION EIGHT
1
How many solutions does the equation x 3 = |x 2| 3 have?
(A) 0
(B) 1
(C) 2
(D) 3
QUESTION NINE
The parametric form of a parabola is (6t, 3t2 ). Its focal length is:
(A)

1
4

(B)

1
4

(C) 3
(D) 3
QUESTION TEN
The polynomial P (x) has degree 4 and the polynomial Q(x) has degree 2. If you divide
P (x) by Q(x), the remainder has degree:
(A) 1
(B) 2
(C) 0 or 1
(D) 0, 1 or 2
End of Section I
Exam continues next page . . .

SGS Trial 2015 . . . . . . . . . . . . . . Form VI Mathematics Extension 1 . . . . . . . . . . . . . . Page 5


SECTION II - Written Response
Answers for this section should be recorded in the booklets provided.
Show all necessary working.
Start a new booklet for each question.

(15 marks) Use a separate writing booklet.

QUESTION ELEVEN

(a) Let A = (1, 4) and B = (5, 5). Find the co-ordinates of the point P which divides
interval AB in the ratio 1 : 2.
(b) Solve the inequation

x
< 2.
2x + 1

Marks

(c) Sketch the graph of y = 2 cos1 (x 1), clearly marking the domain and range.

(d) Differentiate etan x ln x.

(e) Find the coefficient of a3 in the expansion of (2a 1)20 .

(f) Taking x = 14 as a first approximation, use one application of Newtons method to


find a better approximation to 3 sin 2x x = 0.
Give your answer correct to 3 significant figures.

(g) (i) Prove that

sin 2A
= cot A.
1 cos 2A

(ii) Hence find the values of a and b if cot

3
= a + b for integers a and b.
8

Exam continues overleaf . . .

SGS Trial 2015 . . . . . . . . . . . . . . Form VI Mathematics Extension 1 . . . . . . . . . . . . . . Page 6


QUESTION TWELVE

(15 marks) Use a separate writing booklet.

(a) Use the substitution u = tan x to evaluate

sec2 x
dx .
tan2 x + 3

(b) Prove by Mathematical Induction that, for n 1,

Marks

2
3

2 21
3 22
n 2n1
2n
1
1 20
+
+
+ ... +
=
.
23
34
45
(n + 1)(n + 2)
n+2 2

3
1
(c) Find the area bounded by y =
, the line x = 0, the line x =
and the
6
1 9x2
x-axis.
(d) Consider the function y = x2 +
(i) Find

16
.
x

dy
.
dx

(ii) Find the co-ordinates of any stationary points and determine their nature.

(iii) Show that there is a point of inflexion at the x-intercept.

(iv) Sketch the graph y = x2 +

16
, showing the above information.
x

Exam continues next page . . .

SGS Trial 2015 . . . . . . . . . . . . . . Form VI Mathematics Extension 1 . . . . . . . . . . . . . . Page 7


QUESTION THIRTEEN

(15 marks) Use a separate writing booklet.

sin ax
.
x0
x

(a) Find lim

Marks

(b) (i) Show that x


=

d 1 2
( v ).
dx 2

(ii) If v 2 = 24 6x 3x2, find the acceleration of the particle at the particles greatest
displacement from the origin.

(c) Let , and be the roots of the equation x3 px + q = 0. In terms of p and q find
1
1
1
an expression for + + .

(d) Show that tan1 1 + tan1 2 + tan1 3 = .

(e)

N
Q

xm

q
R

120 m

An observer stands at P , 120 metres East of R. A second person is at Q, x metres due


North of R and continues to move North. Let angle RP Q = . Suppose is changing
at 02 radians/minute.
Find the rate at which x is changing when x = 90 metres.

Exam continues overleaf . . .

SGS Trial 2015 . . . . . . . . . . . . . . Form VI Mathematics Extension 1 . . . . . . . . . . . . . . Page 8

(f)

S
C

P
Two diameters AB and CD of a circle, with centre O, are at right angles. Diameter
DC is produced to P and P B cuts the circle again at S.
(i) Prove that AOSP is a cyclic quadrilateral.

(ii) Prove that 6 BCS = 6 SP O.

Exam continues next page . . .

SGS Trial 2015 . . . . . . . . . . . . . . Form VI Mathematics Extension 1 . . . . . . . . . . . . . . Page 9


(15 marks) Use a separate writing booklet.

QUESTION FOURTEEN

Marks

(a) Consider the function f(x) = ln(x x ), for x > 0.


(i) Show that f 0 (x) =

1
(1 ln x).
x2

(ii) Find the range of f(x), giving full reasons.


(b)

y
q

A projectile is fired from the top of a cliff of height h above a horizontal plane with
initial speed V at an angle of elevation . The horizontal range of the projectile is R.
The magnitude of the gravitational acceleration of the projectile is g. Take the origin
at the base of the cliff directly below the launch point of the projectile.
It is known that the vertical and horizontal displacements satisfy
x = V cos t

and

y = h + V sin t 12 gt2 .

(i) Show that the Cartesian equation of motion is


y = h + x tan

gx2
sec2 .
2V 2

V2
V2
tan 2h
= 0.
g
g
 4
 
2
V
V2
V2
2
(iii) Show that R =
+ 2h
R tan
.
g2
g
g
(ii) Show that R2 sec2 2R

(iv) Deduce that the maximum range is

1p 4
V + 2hV 2 g .
g

(v) Show that the angle of elevation satisfies tan =


range.
(vi) Show that tan 2 =

V2
where R1 is the maximum
gR1

R1
.
h

2
2
1
1

2
End of Section II

END OF EXAMINATION

You might also like